You are on page 1of 54

Exam Title : 2019 - Test 19- History & ...

Email : rahulsreedhar8787@gmail.com
Contact :

Note: If the exam is multi-lingual i.e. English and Hindi. Hindi solutions will be
after the completion of English solutions.
QUESTION 1. MTczODc4K1JhaHVsIFNyZWVkaGFyK3JhaHVsc3JlZWRoYXI4Nzg3QGdtYWlsLmNvbStRV
UVTVElPTiAw
Which of the following statements are correct with respect to Gandhiji’s instructions to the
different sections of the society after Quit India resolution was ratified?

1. He advocated Government servants to resign and declare their allegiance to the Congress.

2. He appealed to the peasants to support the local Zamindars unconditionally

3. He declared that the people of all princely states to support their ruler in order to create a
unified mass struggle

Choose the correct code

a) 1 and 2 only
b) 2 and 3 only
c) 1 and 3 only
d) None of the above
Correct Answer: D

Explanation

Solution (d)

Gandhi’s special instructions were spelt out at the Gowalia Tank meeting but not actually
issued. They were directed at various sections of society.

Government servants : Do not resign but declare your allegiance to the Congress .

Soldiers: Do not leave the Army but do not fire on compatriots.

Students: If confident, leave studies.

Peasants: If zamindars are anti-government, pay mutually agreed rent, and if zamindars
are pro-government, do not pay rent.

Princes: Support the masses and accept sovereignty of your people.

Princely states’ people: Support the ruler only if he is anti-government and declare
yourselves to be a part of the Indian nation.

Gandhi followed up with the now-famous exhortation: “Here is a mantra, a short one that I give
you. You may imprint it on your hearts and let every breath of yours give expression to it. The
mantra is: ‘Do or Die’. We shall either free India or die in the attempt; we shall not live to see
the perpetuation of our slavery.

QUESTION 2. MTczODc4K1JhaHVsIFNyZWVkaGFyK3JhaHVsc3JlZWRoYXI4Nzg3QGdtYWlsLmNvbStRV
UVTVElPTiAx

IASbaba
Web: http://ilp.iasbaba.com/ Score:
Email: ilp@iasbaba.com 0.00 / 150
Page 1
Exam Title : 2019 - Test 19- History & ...
Email : rahulsreedhar8787@gmail.com
Contact :

During the course of India’s struggle for freedom Subhas Chandra Bose gave out the philosophy
of samyavada. What does it connote?

a) Combination of Socialism and capitalism


b) Combination of Socialism and communism
c) Combination of Socialism and Fascism
d) Combination of Fascism and racism
Correct Answer: C

Explanation

Solution (c)

Bose expressed the opinion that in India there should be “a synthesis of what modern Europe
calls Socialism and Fascism . We have here the justice, the equality, the love, which is the
basis of Socialism, and combined with that we have the efficiency and the discipline of Fascism
as it stands in Europe today.” He called this ‘samyavada’ . Bose admired discipline and orderly
approach to anything. He admired these qualities in the Fascists of Italy and in the Nazis of
Germany. Indeed, it is clear from his letters that, despite his dislike of colonial power and his
desire to oust the alien British rule from India, he was impressed by the methodical and
systematic approach of the British and their disciplined way of life.

QUESTION 3. MTczODc4K1JhaHVsIFNyZWVkaGFyK3JhaHVsc3JlZWRoYXI4Nzg3QGdtYWlsLmNvbStRV
UVTVElPTiAy
Consider the following statements

1. Gandhiji’s idea of Swaraj laid stress on self-governance through individuals and community
building.

2. Gandhiji advocated a stateless society in which he envisioned a perfect life.

3. Gandhiji was sure that representative democracy could not provide people with justice.

Choose the correct code

a) 1 and 2 only
b) 2 and 3 only
c) 1 and 3 only
d) All of the above
Correct Answer: D

Explanation

Solution (d)

Gandhi’s idealised state, his Ramarajya—a utopia, in fact—did not need a representative
government, a constitution, an army or a police force. Capitalism, communism, exploitation and
religious violence would be absent. Instead, the country was to be modelled on the India of the
past. In many ways, Gandhi’s writings call for a pre-modern, morally-enlightened and apolitical
Indian state. Swaraj lays stress on self-governance through individuals and community

IASbaba
Web: http://ilp.iasbaba.com/ Score:
Email: ilp@iasbaba.com 0.00 / 150
Page 2
Exam Title : 2019 - Test 19- History & ...
Email : rahulsreedhar8787@gmail.com
Contact :

building. “At the individual level Swaraj is vitally connected with the capacity for dispassionate
self-assessment, ceaseless self-purification and growing self-reliance.”

Gandhi said: “I look upon an increase in the power of the state with greatest fear, because
although while apparently doing good by minimising exploitation, it does the greatest harm to
mankind by destroying individuality which is at the root of progress”. He was sceptical of the
party system and sure that representative democracy could not provide people with
justice. He advocated a stateless society in which life becomes perfect.

QUESTION 4. MTczODc4K1JhaHVsIFNyZWVkaGFyK3JhaHVsc3JlZWRoYXI4Nzg3QGdtYWlsLmNvbStRV
UVTVElPTiAz
Consider the following statements with respect to ‘Quit India movement’:

1. The Communists did not join the movement and supported the British war against Germany
as they were attacked in Russia by Nazis.

2. The Muslim League supported the movement and hoped to form a coalition government with
Congress once the British left.

3. The Hindu Mahasabha boycotted the movement

Choose the correct code

a) 1 and 2 only
b) 2 and 3 only
c) 1 and 3 only
d) All of the above
Correct Answer: C

Explanation

Solution (c)

The Quit India movement saw participation on many levels.

· The Communists did not join the movement; in the wake of Russia (where the communists
were in power) being attacked by Nazi Germany, the communists began to support the British
war against Germany and the ‘Imperialist War’ became the ‘People’s War’.

· The Muslim League opposed the movement , fearing that if the British left India at that
time, the minorities would be oppressed by the Hindus.

· The Hindu Mahasabha boycotted the movement.

· The Princely states showed a low-key response.

QUESTION 5. MTczODc4K1JhaHVsIFNyZWVkaGFyK3JhaHVsc3JlZWRoYXI4Nzg3QGdtYWlsLmNvbStRV
UVTVElPTiA0
Consider the following statements about the Formula proposed by C. Rajgopalchari:

IASbaba
Web: http://ilp.iasbaba.com/ Score:
Email: ilp@iasbaba.com 0.00 / 150
Page 3
Exam Title : 2019 - Test 19- History & ...
Email : rahulsreedhar8787@gmail.com
Contact :

1. He demanded the Muslim League to cooperate with Congress in forming a provisional


government at centre.

2. Muslim League was to endorse Congress demand for independence.

3. Decision to form a separate sovereign state for Muslims in the North east and North west
majority areas was to be decided based on the election after the end of the war

Choose the correct code

a) 1 and 2 only
b) 1 and 3 only
c) 2 and 3 only
d) All of the above
Correct Answer: A

Explanation

Solution (a)

C. Rajagopalachari (CR), the veteran Congress leader, prepared a formula for Congress-League
cooperation in 1944. It was a tacit acceptance of the League’s demand for Pakistan. Gandhi
supported the formula. The main points in the CR Plan were:

· Muslim League to endorse Congress demand for independence.

· League to cooperate with Congress in forming a provisional government at centre.

· After the end of the war, the entire population of Muslim majority areas in the North-West and
North-East India to decide by a plebiscite, whether or not to form a separate sovereign state.

· In case of acceptance of partition, agreement to be made jointly for safeguarding defence,


commerce, communications, etc.

· The above terms to be operative only if England transferred full powers to India.

QUESTION 6. MTczODc4K1JhaHVsIFNyZWVkaGFyK3JhaHVsc3JlZWRoYXI4Nzg3QGdtYWlsLmNvbStRV
UVTVElPTiA1
Arrange the following events in their chronological order

1. Wavell plan

2. C.Rajgopalchari formula

3. Desai-liaqat pact

4. Quit India movement

Choose the correct code

a) 1-4-2-3
b) 4-2-3-1

IASbaba
Web: http://ilp.iasbaba.com/ Score:
Email: ilp@iasbaba.com 0.00 / 150
Page 4
Exam Title : 2019 - Test 19- History & ...
Email : rahulsreedhar8787@gmail.com
Contact :

c) 4-3-2-1
d) 3-4-1-2
Correct Answer: B

Explanation

Solution (b)

Quit India Movement (1942)

C. Rajagopalachari Formula (March 1944)

Desai-Liaqat Pact (1945)

· Congress and League nominees to have equal representation in Central Executive

· 20% of seats reserved for minorities

Wavell Plan (Shimla Conference—June 1945)

QUESTION 7. MTczODc4K1JhaHVsIFNyZWVkaGFyK3JhaHVsc3JlZWRoYXI4Nzg3QGdtYWlsLmNvbStRV
UVTVElPTiA2
Consider the following statements with respect to Wavell plan

1. It proposed for more representation for caste Hindus than Muslims in the governor general’s
executive council.

2. It had provision which stated that all members of the executive council were to be Indians
including the governor general.

3. The governor-general was given the power to exercise his veto independently.

Choose the correct code

a) 1 and 2 only
b) 2 and 3 only
c) 1 and 3 only
d) None of the above
Correct Answer: D

Explanation

Solution (d)

The main idea of Wavell plan was to reconstruct the governor-general’s executive council
pending the preparation of a new constitution. For this purpose, a conference was convened by
the viceroy, Lord Wavell, at Shimla in June 1945. The main proposals of the Wavell Plan were as
follows.

· With the exception of the governor-general and the commander-in-chief, all members of the
executive council were to be Indians.

IASbaba
Web: http://ilp.iasbaba.com/ Score:
Email: ilp@iasbaba.com 0.00 / 150
Page 5
Exam Title : 2019 - Test 19- History & ...
Email : rahulsreedhar8787@gmail.com
Contact :

· Caste Hindus and Muslims were to have equal representation.

· The reconstructed council was to function as an interim government within the framework of
the 1935 Act (i.e. not responsible to the Central Assembly).

· The governor-general was to exercise his veto on the advice of ministers.

· Representatives of different parties were to submit a joint list to the viceroy for nominations to
the executive council. If a joint list was not possible, then separate lists were to be submitted.

· Possibilities were to be kept open for negotiations on a new constitution once the war was
finally won.

QUESTION 8. MTczODc4K1JhaHVsIFNyZWVkaGFyK3JhaHVsc3JlZWRoYXI4Nzg3QGdtYWlsLmNvbStRV
UVTVElPTiA3
Who among the following founded the Indian Club of Tokyo and gave lectures on the evils of
Western imperialism?

a) Rashbehari Bose
b) Subhas Chandra Bose
c) Mohan Singh
d) Chittaranjan Das
Correct Answer: A

Explanation

Solution (a)

Rashbehari Bose had fled to Japan in 1915 following the failed revolutionary activities. In Japan,
Rashbehari Bose eventually became a naturalised citizen. He made a lot of effort in getting the
Japanese interested in the Indian independence movement. He became active in Pan-Asian
circles, founded the Indian Club of Tokyo, and gave lectures on the evils of Western
imperialism.

Very early itself he was impressed by Subhas Chandra Bose. When the Indian National Army
was formed by Mohan Singh in Singapore, Rashbehari Bose was greatly excited and left Tokyo
for Southeast Asia. It was at a conference in Bangkok (also under Japanese occupation at the
time) that it was decided to place the INA under an Indian Independence League whose
chairman would be Rashbehari Bose himself. He had created the League in 1942 in Tokyo.

QUESTION 9. MTczODc4K1JhaHVsIFNyZWVkaGFyK3JhaHVsc3JlZWRoYXI4Nzg3QGdtYWlsLmNvbStRV
UVTVElPTiA4
Which of the following statements are correct about Subhas Chandra Bose

1. He was the first person to call Gandhi as Father of Nation.

2. The famous slogan "Give me blood, I will give you freedom" was given by him

3. He gave the war cry, ‘Jai Hind’ from the Free India Centre, Germany.

IASbaba
Web: http://ilp.iasbaba.com/ Score:
Email: ilp@iasbaba.com 0.00 / 150
Page 6
Exam Title : 2019 - Test 19- History & ...
Email : rahulsreedhar8787@gmail.com
Contact :

Choose the correct code

a) 1 and 2 only
b) 2 and 3 only
c) 1 and 3 only
d) All of the above
Correct Answer: D

Explanation

Solution (d)

Bose met Hitler and with his help, the ‘Freedom Army’ (Mukti Sena) was formed which
consisted of all the prisoners of war of Indian origin captured by Germany and Italy. Dresden,
Germany was made the office of the Freedom Army. Bose came to be called ‘Netaji’ by the
people of Germany. He gave the famous slogan, ‘Jai Hind’ from the Free India Centre, Germany.

On October 21, 1943, Subhas Bose formed the Provisional Government for Free India at
Singapore with H.C. Chatterjee (Finance portfolio), M.A. Aiyar (Broadcasting), Lakshmi
Swaminathan (Women Department), etc. The famous slogan—“Give me blood, I will give you
freedom” was given in Malaya.

On July 6, 1944, Subhas Bose addressed Mahatma Gandhi as ‘Father of Nation’—from the Azad
Hind Radio (the first person to call Gandhi, ‘Father of Nation’).

QUESTION 10. MTczODc4K1JhaHVsIFNyZWVkaGFyK3JhaHVsc3JlZWRoYXI4Nzg3QGdtYWlsLmNvbStRV


UVTVElPTiA5
British government changed its attitude towards India after the war by convening constituent
assembly. Which of the following are correct reasons for it?

1. The end of the War resulted in a change in balance of global power with UK losing its power
to US and USSR

2. The new Labour government in UK was more sympathetic to Indian demands

3. There was a wave of socialist radical governments throughout Europe

Choose the correct code

a) 1 and 2 only
b) 2 and 3 only
c) 1 and 3 only
d) All of the above
Correct Answer: D

Explanation

Solution (d)

A change in British Government’s attitude after the war was due to

IASbaba
Web: http://ilp.iasbaba.com/ Score:
Email: ilp@iasbaba.com 0.00 / 150
Page 7
Exam Title : 2019 - Test 19- History & ...
Email : rahulsreedhar8787@gmail.com
Contact :

· Change in global power equations; UK no longer a power

· Labour government sympathetic to India

· Tired British soldiers and shattered British economy

· Anti-imperialist wave throughout Asia

QUESTION 11. MTczODc4K1JhaHVsIFNyZWVkaGFyK3JhaHVsc3JlZWRoYXI4Nzg3QGdtYWlsLmNvbStRVUVTV


ElPTiAxMA==
Which of the following are correct reasons for the revolt by the ratings of Royal Indian Navy on
INS Talwar?

1. It was because of racial discrimination suffered by Indian soldiers in contrast with white
soldiers

2. Food given to the Indian soldiers was unpalatable

3. It was directed against the use of Indian troops in Indonesia

Choose the correct code

a) 1 and 2 only
b) 2 and 3 only
c) 1 and 3 only
d) All of the above
Correct Answer: D

Explanation

Solution (d)

Rebellion by Naval Ratings On February 18, 1946 some 1100 Royal Indian Navy (RIN) ratings of
HMIS Talwar went on a strike to protest against

· racial discrimination (demanding equal pay for Indian and white soldiers)

· unpalatable food

· abuse by superior officers

· arrest of a rating for scrawling ‘Quit India’ on HMIS Talwar

· INA trials

· Use of Indian troops in Indonesia, demanding their withdrawal.

The rebellious ratings hoisted the tricolour, crescent, and the hammer and sickle flags on the
mast of the rebel fleet. Other ratings soon joined and they went around Bombay in Lorries
holding Congress flags threatening Europeans and policemen. Crowds brought food to the
ratings and shopkeepers invited them to take whatever they needed.

IASbaba
Web: http://ilp.iasbaba.com/ Score:
Email: ilp@iasbaba.com 0.00 / 150
Page 8
Exam Title : 2019 - Test 19- History & ...
Email : rahulsreedhar8787@gmail.com
Contact :

QUESTION 12. MTczODc4K1JhaHVsIFNyZWVkaGFyK3JhaHVsc3JlZWRoYXI4Nzg3QGdtYWlsLmNvbStRVUVTV


lPTiAxMQ==
Consider the following statements about Cabinet mission plan

1. It was aimed at negotiating a peaceful transfer of power to India

2. It rejected the demand for the formation of a full-fledged Pakistan state

3. It proposed that a constituent assembly was to be elected by provincial assemblies by


proportional representation

Choose the correct code

a) 1 and 2 only
b) 2 and 3 only
c) 1 and 3 only
d) All of the above
Correct Answer: D

Explanation

Solution (d)

The Attlee government announced in February 1946 the decision to send a high-powered
mission of three British cabinet members (Pethick Lawrence, Secretary of State for India;
Stafford Cripps, President of the Board of Trade; and A.V. Alexander, First Lord of Admiralty) to
India to find out ways and means for a negotiated, peaceful transfer of power to India.

Proposals of Cabinet mission

· Rejection of Pakistan

· Grouping of existing assemblies into three sections A, B, C

· Three-tier executive and legislature at province, princely states and union level

· Provincial assemblies to elect a constituent assembly

· Common centre for defence, communications, external affairs

· Provinces to have autonomy and residual powers

· Princely states free to have an arrangement with the successor government or the British
Government

· In future, a province free to come out of the section or the union

QUESTION 13. MTczODc4K1JhaHVsIFNyZWVkaGFyK3JhaHVsc3JlZWRoYXI4Nzg3QGdtYWlsLmNvbStRVUVTV


ElPTiAxMg==

IASbaba
Web: http://ilp.iasbaba.com/ Score:
Email: ilp@iasbaba.com 0.00 / 150
Page 9
Exam Title : 2019 - Test 19- History & ...
Email : rahulsreedhar8787@gmail.com
Contact :

Which of the following were the reasons given by the Cabinet mission for the non-formation of
Pakistan?

1. The Pakistan so formed would include a large non-Muslim population

2. It was assumed that deep-seated regional ties would be disturbed if Bengal and Punjab were
partitioned

3. The partition would entail economic and administrative problems between east and west
Pakistan

Choose the correct code

a) 1 and 2 only
b) 2 and 3 only
c) 1 and 3 only
d) All of the above
Correct Answer: D

Explanation

Solution (d)

Cabinet mission rejected the demand for the formation of a full-fledged Pakistan state because

· The Pakistan so formed would include a large non-Muslim population—38 per cent in the
North-West and 48 per cent in the North-East;

· The very principle of communal self-determination would claim separation of Hindu-majority


western Bengal and Sikh- and Hindu-dominated Ambala and Jullundur divisions of Punjab
(already some Sikh leaders were demanding a separate state if the country was partitioned);

· Deep-seated regional ties would be disturbed if Bengal and Punjab were partitioned;

· Partition would entail economic and administrative problems, for instance, the problem of
communication between the western and eastern parts of Pakistan; and

· The division of the armed forces would be dangerous.

QUESTION 14. MTczODc4K1JhaHVsIFNyZWVkaGFyK3JhaHVsc3JlZWRoYXI4Nzg3QGdtYWlsLmNvbStRVUVTV


lPTiAxMw==
Consider the following with respect to Attlee’s Statement

1. It said the transfer of power would take place after Indian politicians agree about the time on
the constitution.

2. It gave a call that British powers and obligations vis-a-vis the princely states would lapse with
transfer of power

Choose the correct code

a) 1 only

IASbaba
Web: http://ilp.iasbaba.com/ Score:
Email: ilp@iasbaba.com 0.00 / 150
Page 10
Exam Title : 2019 - Test 19- History & ...
Email : rahulsreedhar8787@gmail.com
Contact :

b) 2 only
c) Both 1 and 2
d) None of the above
Correct Answer: B
Explanation

Solution (b)

Main Points of Attlee’s


Statement

· A deadline of June 30, 1948 was fixed for transfer of power even if the Indian politicians had
not agreed by that time on the constitution.

· The British would relinquish power either to some form of central government or in some
areas to the existing provincial governments if the constituent assembly was not fully
representative, i.e., if the Muslim majority provinces did not join.

· British powers and obligations vis-a-vis the princely states would lapse with transfer of power,
but these would not be transferred to any successor government in British India.

· Mountbatten would replace Wavell as the viceroy.

The statement contained clear hints of partition and even Balkanisation of the country into
numerous states and was, in essence, a reversion of the Cripps’s offer

QUESTION 15. MTczODc4K1JhaHVsIFNyZWVkaGFyK3JhaHVsc3JlZWRoYXI4Nzg3QGdtYWlsLmNvbStRVUVTV


ElPTiAxNA==
Which of the following are correct as to why the date was fixed by British Government for
withdrawal from India?

1. The government was keen to avert the developing constitutional crisis

2. The government hoped to convince the Indians of British sincerity

3. The government hoped that a fixed date would shock the parties into an agreement on the
main question

Choose the correct code

a) 1 and 2 only
b) 2 and 3 only
c) 1 and 3 only
d) All of the above
Correct Answer: D

Explanation

Solution (d)

Reasons as to Why a Date Fixed by Government for withdrawal

IASbaba
Web: http://ilp.iasbaba.com/ Score:
Email: ilp@iasbaba.com 0.00 / 150
Page 11
Exam Title : 2019 - Test 19- History & ...
Email : rahulsreedhar8787@gmail.com
Contact :

· The government hoped that a fixed date would shock the parties into an agreement on the
main question.

· The government was keen to avert the developing constitutional crisis.

· The government hoped to convince the Indians of British sincerity.

· The truth in Wavell’s assessment could no longer be denied—that an irreversible decline of the
government’s authority had taken place.

QUESTION 16. MTczODc4K1JhaHVsIFNyZWVkaGFyK3JhaHVsc3JlZWRoYXI4Nzg3QGdtYWlsLmNvbStRVUVTV


ElPTiAxNQ==
Which of the following are correct about Congress accepting Dominion Status from British?

1. It was thought to ensure a peaceful and quick transfer of power to Indians

2. It would give authority for the Congress to check the explosive situation

Choose the correct code

a) 1 only
b) 2 only
c) Both 1 and 2
d) None of the above
Correct Answer: C

Explanation

Solution (c)

The Congress was willing to accept dominion status despite its being against the Lahore
Congress (1929) spirit because

· It would ensure a peaceful and quick transfer of power

· It was more important for the Congress to assume authority to check the explosive situation

· It would allow for some much needed continuity in the bureaucracy and the army.

QUESTION 17. MTczODc4K1JhaHVsIFNyZWVkaGFyK3JhaHVsc3JlZWRoYXI4Nzg3QGdtYWlsLmNvbStRVUVTV


ElPTiAxNg==
Consider the following statements about Indian Independence Act

1. It was based on the recommendations of Mountbatten Plan

2. The Act provided for the creation of two independent dominions of India and Pakistan

3. It provided that the governments of the two dominions were to be carried on in accordance
with the Government of India Act, 1935 during the transition period

IASbaba
Web: http://ilp.iasbaba.com/ Score:
Email: ilp@iasbaba.com 0.00 / 150
Page 12
Exam Title : 2019 - Test 19- History & ...
Email : rahulsreedhar8787@gmail.com
Contact :

Choose the correct code

a) 1 and 2 only
b) 2 and 3 only
c) 1 and 3 only
d) All of the above
Correct Answer: D

Explanation

Solution (d)

On July 5, 1947 the British Parliament passed the Indian Independence Act which was based
on the Mountbatten Plan , and the Act got royal assent on June 18, 1947. The Act was
implemented on August 15, 1947.

The Act provided for the creation of two independent dominions of India and Pakistan with
effect from August 15, 1947. Each dominion was to have a governor-general to be responsible
for the effective operation of the Act. The constituent assembly of the each new dominion was
to exercise the powers of the legislature of that dominion, and the existing Central Legislative
Assembly and the Council of States were to be automatically dissolved. For the transitional
period, i.e., till a new constitution was adopted by each dominion, the governments of the two
dominions were to be carried on in accordance with the Government of India Act, 1935.

QUESTION 18. MTczODc4K1JhaHVsIFNyZWVkaGFyK3JhaHVsc3JlZWRoYXI4Nzg3QGdtYWlsLmNvbStRVUVTV


ElPTiAxNw==
Consider the following statements about the Plan Balkan

1. It was proposed by lord Mountbatten

2. It proposed to transfer power of separate provinces or confederation of provinces, with the


Bengal and Punjab Assemblies being given the options to vote for partition of their provinces

Choose the correct code

a) 1 only
b) 2 only
c) Both 1 and 2
d) None of the above
Correct Answer: C

Explanation

Solution (c)

Plan Balkan

Between March and May of 1947, Mountbatten decided that the Cabinet Mission Plan had
become untenable and formulated an alternative plan. This plan envisaged the transfer of
power to separate provinces (or to a confederation, if formed before the transfer), with Punjab

IASbaba
Web: http://ilp.iasbaba.com/ Score:
Email: ilp@iasbaba.com 0.00 / 150
Page 13
Exam Title : 2019 - Test 19- History & ...
Email : rahulsreedhar8787@gmail.com
Contact :

and Bengal given the option to vote for partition of their provinces. The various units thus
formed along with the princely states (rendered independent by lapse of paramountcy) would
have the option of joining India or Pakistan or remaining separate. The plan was quickly
abandoned after Nehru reacted violently to it.

Nehru rejected the plan right away and told him that this plan would invite Balkanisation of
India and would provoke conflict and violence.

QUESTION 19. MTczODc4K1JhaHVsIFNyZWVkaGFyK3JhaHVsc3JlZWRoYXI4Nzg3QGdtYWlsLmNvbStRVUVTV


ElPTiAxOA==
Which of the following were touted to be the long term challenges faced by India after it
became Independent?

1. Economic development

2. Framing of the Constitution for India

3. Abolition of feudal set up in agriculture

4. National integration

5. Poverty alleviation

Choose the correct code

a) 1,2,3 and 4 only


b) 1,4 and 5 only
c) 2,3,4 and 5 only
d) All of the above
Correct Answer: B

Explanation

Solution (b)

Independent India, had to face several challenges which among the following include

Immediate Challenges —territorial and administrative integration of princely states,


communal riots, rehabilitation of nearly 60 lakh refugees migrated from Pakistan, protection of
Muslims living in India as well as those going to Pakistan from communal gangs, need to avoid
war with Pakistan, Communist insurgency, etc.

Medium Term Challenges —framing of the Constitution for India, building of a


representative, democratic and civil libertarian political order, elections, and abolition of feudal
set up in agriculture, etc.

Long Term Challenges —national integration, economic development, poverty alleviation, etc.

QUESTION 20. MTczODc4K1JhaHVsIFNyZWVkaGFyK3JhaHVsc3JlZWRoYXI4Nzg3QGdtYWlsLmNvbStRVUVTV


ElPTiAxOQ==

IASbaba
Web: http://ilp.iasbaba.com/ Score:
Email: ilp@iasbaba.com 0.00 / 150
Page 14
Exam Title : 2019 - Test 19- History & ...
Email : rahulsreedhar8787@gmail.com
Contact :

Consider the following statements about Delhi pact on minorities

1. It was concluded between Jawahar Lal Nehru and Mohammad Ali Jinnah

2. As per this pact the refugees from both the countries were allowed to return unmolested to
dispose off their property, abducted women and looted property were to be returned

Choose the correct code

a) 1 only
b) 2 only
c) Both 1 and 2
d) None of the above
Correct Answer: B

Explanation

Solution (b)

Delhi Pact, also called Nehru-Liaqat Pact, pact made on April 8, 1950, following the escalation
of tension between India and Pakistan in East Pakistan (now Bangladesh) after economic
relations between the two countries had been severed in December 1949. An estimated one
million people—Hindus from East Pakistan and Muslims from West Bengal—crossed the borders
during 1950.

The treaty was signed in New Delhi by the Prime Minister Jawahar Lal Nehru and the Prime
Minister Liaquat Ali Khan on April 8, 1950.

As per the provisions of this pact refugees were allowed to return unmolested to dispose of
their property, abducted women and looted property were to be returned, forced conversions
were unrecognized, and minority rights were confirmed. Minority commissions were
established to implement these terms, and confidence was in fact restored for a time; however,
in the months following the pact, more than a million additional refugees migrated to West
Bengal.

QUESTION 21. MTczODc4K1JhaHVsIFNyZWVkaGFyK3JhaHVsc3JlZWRoYXI4Nzg3QGdtYWlsLmNvbStRVUVTV


ElPTiAyMA==
Which of the following are correctly matched?

State Integration into the Indian Union

1. Hyderabad A) By police action

2. Junagarh B) By Plebiscite

3. Jammu and Kashmir C) Instrument of Accession

Choose the correct code

a) 1 and 2 only
b) 2 and 3 only

IASbaba
Web: http://ilp.iasbaba.com/ Score:
Email: ilp@iasbaba.com 0.00 / 150
Page 15
Exam Title : 2019 - Test 19- History & ...
Email : rahulsreedhar8787@gmail.com
Contact :

c) 1 and 3 only
d) All of the above
Correct Answer: D
Explanation

Solution (d)

1. Junagarh -The Muslim Nawab wanted to join Pakistan but a Hindu majority population
wanted to join the Indian Union. In the face of repressive attitude of the nawab, there was a ple
biscite which decided in favour of India.

2. Hyderabad –It wanted a sovereign status. It signed a Standstill Agreement with India in
November 1947. Indian troops withdrew and the Nizam’s police and storm troopers
(Razakkars) took over. The Nizam wanted an outlet to the sea (Goa). The violence and supply of
foreign arms prompted Indian troops to move in again in 1948— described as “a police action
to restore law and order”. Hyderabad acceded in November 1949.

3. Kashmir The state of Jammu and Kashmir had a Hindu prince and a Muslim majority
population. The prince envisaged a sovereign status for the state and was reluctant to accede to
either of the dominions. As he procrastinated, the newly established state of Pakistan sent its
forces behind a front of tribal militia and moved menacingly towards Srinagar. It was now that
the prince was forced to sign an Instrument of Accession (October 1947) with the Indian
Union, endorsed by the popular leader Sheikh Abdullah.

QUESTION 22. MTczODc4K1JhaHVsIFNyZWVkaGFyK3JhaHVsc3JlZWRoYXI4Nzg3QGdtYWlsLmNvbStRVUVTV


ElPTiAyMQ==
Consider the following statements about Objectives Resolution

1. It was introduced by Jawaharlal Nehru as a response to the India Independence act of 1947

2. It proclaimed India to be an “Independent Sovereign Socialist Republic”

3. It contained the defining ideals of the Constitution of Independent India

Choose the correct code

a) 1 and 2 only
b) 2 only
c) 3 only
d) 1 and 3 only
Correct Answer: C

Explanation

Solution (c)

On 13 December 1946, Jawaharlal Nehru introduced the “Objectives Resolution” in the


Constituent Assembly. It was a momentous resolution that outlined the defining ideals of the
Constitution of Independent India, and provided the framework within which the work of
constitution-making was to proceed.

IASbaba
Web: http://ilp.iasbaba.com/ Score:
Email: ilp@iasbaba.com 0.00 / 150
Page 16
Exam Title : 2019 - Test 19- History & ...
Email : rahulsreedhar8787@gmail.com
Contact :

It proclaimed India to be an “Independent Sovereign Republic” , guaranteed its citizens


justice, equality and freedom, and assured that “adequate safeguards shall be provided for
minorities, backward and tribal areas, and Depressed and Other Backward Classes … ” After
outlining these objectives, Nehru placed the Indian experiment in a broad historical
perspective. As he spoke, he said, his mind went back to the historic efforts in the past to
produce such documents of rights.

QUESTION 23. MTczODc4K1JhaHVsIFNyZWVkaGFyK3JhaHVsc3JlZWRoYXI4Nzg3QGdtYWlsLmNvbStRVUVTV


ElPTiAyMg==
Consider the following statements about the criteria of Non-alignment movement

1. A country should have consistently supported national freedom movements in other


countries.

2. It should not be a member of multi-lateral military alliances concluded in the context of


super-power conflicts.

3. A country should follow a policy based on peaceful co-existence with the other non-alignment
members.

Choose the correct code

a) 1 and 2 only
b) 2 and 3 only
c) 1 and 3 only
d) All of the above
Correct Answer: A

Explanation

Solution (a)

The Preparatory Committee of the first non-aligned conference laid down the following five
criteria of non-alignment:

· A country should follow an independent policy based on peaceful co-existence and non-
alignment.

· It should have consistently supported national freedom movements in other countries.

· It should not be a member of multi-lateral military alliances concluded in the context of super-
power conflicts.

· If it has conceded military bases, these concessions should not have been made in the context
of super-power conflicts.

· If it is a member of bilateral or regional defence arrangements, this should not be in the


context of super-power politics.

IASbaba
Web: http://ilp.iasbaba.com/ Score:
Email: ilp@iasbaba.com 0.00 / 150
Page 17
Exam Title : 2019 - Test 19- History & ...
Email : rahulsreedhar8787@gmail.com
Contact :

QUESTION 24. MTczODc4K1JhaHVsIFNyZWVkaGFyK3JhaHVsc3JlZWRoYXI4Nzg3QGdtYWlsLmNvbStRVUVTV


ElPTiAyMw==
Which of the following are correctly matched?

Founder Political party

1. Dr.B.R. Ambedkar A) Republican party of India

2. J.B. Kripalani B) Kisan Mazdoor Praja Party

3. Shyam Prasad Mukherjee C) Bharatiya Jana Sangh

Choose the correct code

a) 1 and 2 only
b) 2 and 3 only
c) 1 and 3 only
d) All of the above
Correct Answer: D

Explanation

Solution (d)

After Independence, though it was generally accepted that the Congress had the largest
following, various other political strands in India were also beginning to take shape. Just before
the first elections, Shyam Prasad Mukherjee (industries minister under Nehru) broke
away to set up the Bharatiya Jana Sangh (a proto-BJP) in October 1951.

Dr. B. R. Ambedkar revived the Scheduled Castes Federation (which was later named the
Republican Party).

Another high-profile Congress leader, J. B. (Acharya) Kripalani, founded the Kisan


Mazdoor Praja Party . Ram Manohar Lohia and Jaya Prakash Narayan were the forces behind
the Socialist Party.

QUESTION 25. MTczODc4K1JhaHVsIFNyZWVkaGFyK3JhaHVsc3JlZWRoYXI4Nzg3QGdtYWlsLmNvbStRVUVTV


ElPTiAyNA==
Which of the following committee/s supported the reorganisation of states on linguistic basis?

1. S.K. Dhar Committee

2. JVP Committee

3. Fazl Ali committee

Choose the correct code

a) 1 and 2 only
b) 2 only
c) 3 only

IASbaba
Web: http://ilp.iasbaba.com/ Score:
Email: ilp@iasbaba.com 0.00 / 150
Page 18
Exam Title : 2019 - Test 19- History & ...
Email : rahulsreedhar8787@gmail.com
Contact :

d) 2 and 3 only
Correct Answer: C

Explanation

Solution (c)

The Constituent Assembly, in June 1948, appointed the Linguistic Provinces Commission,
headed by Justice S.K. Dhar, to enquire into the need of linguistic provinces. The Dhar
Commission, however, opposed such a move in the interest of national integration. Conse
quently the Constituent Assembly decided not to include the linguistic principle in the
constitution.

In December 1948, to pacify the vocal votaries of linguistic states, the Congress appointed a
committee (JVP), with Jawaharlal Nehru, Vallabhbhai Patel and Pattabhi Sitaramayya as its
members. Its report which is known as the JVP Report—also went against the creation of
linguistic states in the interests of national unity .

Due to incessant efforts of Potti Sriramulu who died after fasting for fifty-six days in favour of
separate state of Andhra, Nehru government appointed the States Reorganisation Commission
(SRC) in August 1953.

The commission, comprising Justice Fazl Ali, K.M. Panikkar and Hridaynath Kunzru as
members, submitted its report in October 1955; It supported the reorganisation of states
on linguistic basis and its recommendations were accepted with some modifications and
implemented quickly.

QUESTION 26. MTczODc4K1JhaHVsIFNyZWVkaGFyK3JhaHVsc3JlZWRoYXI4Nzg3QGdtYWlsLmNvbStRVUVTV


ElPTiAyNQ==
Where did Subhash Chandra Bose assumed the leadership of INA?

a) Japan
b) Singapore
c) Germany
d) Burma
Correct Answer: B

Explanation

Solution (b)

On 4 July 1943, two days after reaching Singapore, Bose assumed the leadership of the IIL and
the INA, from Capt. Mohan Singh in a ceremony at Cathay Building. Bose's influence was
notable. His appeal re-invigorated the INA, which had previously consisted mainly of prisoners
of war: it also attracted Indian expatriates in South Asia. He famously proclaimed:

“Give me blood! I will give you freedom”

IASbaba
Web: http://ilp.iasbaba.com/ Score:
Email: ilp@iasbaba.com 0.00 / 150
Page 19
Exam Title : 2019 - Test 19- History & ...
Email : rahulsreedhar8787@gmail.com
Contact :

QUESTION 27. MTczODc4K1JhaHVsIFNyZWVkaGFyK3JhaHVsc3JlZWRoYXI4Nzg3QGdtYWlsLmNvbStRVUVTV


ElPTiAyNg==
Government of India has released a coin for Rs75 to commemorate the first hoisting of
Tricolour Flag by Netaji Subhash Chandra Bose. Where did he hoist the flag?

a) Andaman and Nicobar


b) Singapore
c) Nagaland
d) Kolkata
Correct Answer: A

Explanation

Solution (a)

The government announced it will release a Rs 75 commemorative coin on the occasion of 75th
anniversary of hoisting of Tricolour by Subhash Chandra Bose for the first time at Port Blair
(Andaman and Nicobar Islands).

The 35 gram coin will be composed of 50% silver, 40% copper and 5% each of Nickel and Zinc.

QUESTION 28. MTczODc4K1JhaHVsIFNyZWVkaGFyK3JhaHVsc3JlZWRoYXI4Nzg3QGdtYWlsLmNvbStRVUVTV


ElPTiAyNw==
In India, ‘Mixed Economy’ was envisaged for the first time in –

a) Nehru Report
b) First Five Year Plan
c) Second Five year Plan
d) Bombay Plan
Correct Answer: D

Explanation

Solution (d)

‘ Bombay Plan’ , authored by a group of Indian industrialists and technocrats in 1944, was
meant as a fifteen year investment plan for India. The strategy of the government’s five year
plans after independence was very similar to the Bombay Plan’s. First three five year plans had
almost the same sectoral outlay pattern and together they can be described as a scaled down
version of the Bombay Plan. The Plan remained at the centre of news and interest for more
than a year. But thereafter it was as completely forgotten as it was at the centre of interest
previously.

A key principle of the Bombay Plan was that the economy could not grow without government
intervention and regulation. Under the assumption that the fledgling Indian industries would
not be able to compete in a free-market economy, the Plan proposed that the future government
protect indigenous industries against foreign competition in local markets. Other salient points
of the Bombay plan were an active role by government in deficit financing and planning
equitable growth, a transition from an agrarian to an industrialized society, and—in the event
that the private sector could not immediately do so—the establishment of critical industries as

IASbaba
Web: http://ilp.iasbaba.com/ Score:
Email: ilp@iasbaba.com 0.00 / 150
Page 20
Exam Title : 2019 - Test 19- History & ...
Email : rahulsreedhar8787@gmail.com
Contact :

public sector enterprises while simultaneously ensuring a market for the output through
planned purchases.

QUESTION 29. MTczODc4K1JhaHVsIFNyZWVkaGFyK3JhaHVsc3JlZWRoYXI4Nzg3QGdtYWlsLmNvbStRVUVTV


ElPTiAyOA==
Which of the following points were including in the Panchsheel agreement of Peaceful
coexistence with China?

1. Mutual respect for each other’s sovereignty and territorial and territorial integrity

2. Non – Aggression

3. No-interference in the external affairs of each other

4. Non Alignment with either of the super power

5. Equality and mutual benefit

Select the code from following:

a) 1,2 and 3
b) 2,3 and 4
c) 1,2 and 5
d) All of the above
Correct Answer: C

Explanation

Solution (c)

In the Panchsheel treaty, the leaders of China and India outlined their policy of Panchsheel and
defined it through five ideological principles. Those principles are:

• Mutual respect for each other's territorial integrity and sovereignty


• Mutual non-aggression
• Mutual non-interference
• Equality and cooperation for mutual benefit
• Peaceful coexistence

According to Nehru and Zhou, China and India's relationship would be one of respect and
peace. Basically, each nation agreed not to mess with the internal affairs of the other and
committed them to pursuing peaceful solutions to any conflicts that might arise between them.

QUESTION 30. MTczODc4K1JhaHVsIFNyZWVkaGFyK3JhaHVsc3JlZWRoYXI4Nzg3QGdtYWlsLmNvbStRVUVTV


ElPTiAyOQ==
Who of the following personalities designed the National Flag of India adopted by the
Constituent Assembly on 21 st July, 1947?

a) Pingali Venkayya

IASbaba
Web: http://ilp.iasbaba.com/ Score:
Email: ilp@iasbaba.com 0.00 / 150
Page 21
Exam Title : 2019 - Test 19- History & ...
Email : rahulsreedhar8787@gmail.com
Contact :

b) Badruddin Tayyabji
c) Lakshmi Sehgal
d) Hasrat Mohani
Correct Answer: A

Explanation

Solution (a)

The National Flag of India is a horizontal rectangular tricolour of India saffron, white and India
green; with the Ashoka Chakra, a 24-spoke wheel, in navy blue at its centre. It was adopted in
its present form during a meeting of the Constituent Assembly held on 22 July 1947, and it
became the official flag of the Dominion of India on 15 August 1947. The flag was subsequently
retained as that of the Republic of India. In India, the term "tricolour" almost always refers to
the Indian national flag. The flag is based on the Swaraj flag, a flag of the Indian National
Congress designed by Pingali Venkayya.

QUESTION 31. MTczODc4K1JhaHVsIFNyZWVkaGFyK3JhaHVsc3JlZWRoYXI4Nzg3QGdtYWlsLmNvbStRVUVTV


ElPTiAzMA==
Who of the following was not one of the founders of the Non Aligned Movement at Yugoslavia?

a) Pt Nehru
b) Josip Broz Tito
c) Suharto
d) Abdel Nasser
Correct Answer: C

Explanation

Solution (c)

The founding fathers of the Non-Aligned Movement were Josip Broz Tito of Socialist
Yugoslavia, Jawaharlal Nehru of India, Sukarno of Indonesia, Gamal Abdel Nasser of Egypt
and Kwame Nkrumah of Ghana. Their actions were known as 'The Initiative of Five'.

The Non-Aligned Movement as an organization was founded on the Brijuni islands in Yugoslavia
in 1956, and was formalized by signing the Declaration of Brijuni on 19 July 1956. The
Declaration was signed by Yugoslavia's president, Josip Broz Tito, India's first prime minister
Jawaharlal Nehru and Egypt's second president, Gamal Abdel Nasser. One of the quotations
within the Declaration is "Peace cannot be achieved with separation, but with the aspiration
towards collective security in global terms and expansion of freedom, as well as terminating the
domination of one country over another". According to Rejaul Karim Laskar, an ideologue of the
Congress party which ruled India for most part of the Cold War years, the Non-Aligned
Movement arose from the desire of Jawaharlal Nehru and other leaders of the newly
independent countries of the third world to guard their independence "in face of complex
international situation demanding allegiance to either two warring superpowers"

IASbaba
Web: http://ilp.iasbaba.com/ Score:
Email: ilp@iasbaba.com 0.00 / 150
Page 22
Exam Title : 2019 - Test 19- History & ...
Email : rahulsreedhar8787@gmail.com
Contact :

QUESTION 32. MTczODc4K1JhaHVsIFNyZWVkaGFyK3JhaHVsc3JlZWRoYXI4Nzg3QGdtYWlsLmNvbStRVUVTV


ElPTiAzMQ==
Consider the following statements:

1. He is considered to be father of Statistics in India.

2. He was the planner of India’s first two five year plans after independence.

3. He analysed 60 years of data regarding the floods in Odisha and published his findings in
1926, which three decades later laid the foundation for the construction of the Hirakud dam on
the Mahanadi River.

Above points are referring to which of the following personalities?

a) Harrod Domer
b) Mahalinobis
c) V P Menon
d) Sardar Patel
Correct Answer: B

Explanation

Solution (b)

This year 125 th Birth Anniversary of Mahalinobis was celebrated. Following are some
important points about him.

· Mahalanobis took up a teaching position in the physics department of Presidency College,


Calcutta, and set-up a statistical laboratory in the early 1920s. Here, he combined his interest
in statistics with other disciplines like economics, anthropology and economics to develop
unique ways to estimate household consumption, crop yields and population.

· Mahalanobis treated statistics as a “new technology for increasing the efficiency of human
effort in the wildest sense”. Most famously, he analysed 60 years of data regarding the floods in
Odisha and published his findings in 1926, which three decades later laid the foundation for the
construction of the Hirakud dam on the Mahanadi river.

· In 1931, Mahalanobis established the famous Indian Statistical Institute—a specialised


institute to facilitate research and learning of statistics within the premises of Presidency
College. Two years later, he set up Sankhya, the Indian Journal of Statistics.

· He Came up with the famous “Mahalanobis Distance”, a measurement often used in studies of
population distribution.

· The National Same Survey (NSS) was his brainchild. Launched in 1950, it was a herculean
task towards providing a comprehensive representation of India’s economy.

QUESTION 33. MTczODc4K1JhaHVsIFNyZWVkaGFyK3JhaHVsc3JlZWRoYXI4Nzg3QGdtYWlsLmNvbStRVUVTV


ElPTiAzMg==
What is the correct chronological order of the following events after independence?

1. Assassination of Mahatma Gandhi

IASbaba
Web: http://ilp.iasbaba.com/ Score:
Email: ilp@iasbaba.com 0.00 / 150
Page 23
Exam Title : 2019 - Test 19- History & ...
Email : rahulsreedhar8787@gmail.com
Contact :

2. Presentation of the First Railway Budget

3. Accession of Kashmir to Indian Union

4. Nationalisation of the RBI

Select the code from following:

a) 1,2,3 and 4
b) 2,4,3 and 1
c) 3,2,1 and 4
d) 4,2,1 and 3
Correct Answer: C
Explanation

Solution (c)

· The Instrument of Accession is a legal document executed by Maharaja Hari Singh, ruler of
the princely state of Jammu and Kashmir, on 26 October 1947 .

· John Mathai presented the first Railway Budget for independent India in November
1947 .

· Mahatma Gandhi was assassinated on 30 January 1948 in the compound of Birla House

· The Reserve Bank of India was nationalised with effect from 1st January, 1949 on the basis of
the Reserve Bank of India (Transfer to Public Ownership) Act, 1948.

QUESTION 34. MTczODc4K1JhaHVsIFNyZWVkaGFyK3JhaHVsc3JlZWRoYXI4Nzg3QGdtYWlsLmNvbStRVUVTV


ElPTiAzMw==
Razakars were the armed militia of which of the following Princely States?

a) Bhopal
b) Kashmir
c) Junagarh
d) Hyderabad
Correct Answer: D

Explanation

Solution (d)

The Razakars were a private militia organised by Qasim Razvi to support the rule of Nizam
Osman Ali Khan, Asaf Jah VII and resist the integration of Hyderabad State into the Dominion of
India. They also attempted to make the Nizam accede his princely state to Pakistan instead of
India.

They were terrorizing the supporters of Union of India (especially Hindus). Indian government
had to take a strict against them by sending Indian army in Hyderabad. The Nizam of Hydrabad
surrendered peacefully and signed the instrument of accession.

IASbaba
Web: http://ilp.iasbaba.com/ Score:
Email: ilp@iasbaba.com 0.00 / 150
Page 24
Exam Title : 2019 - Test 19- History & ...
Email : rahulsreedhar8787@gmail.com
Contact :

QUESTION 35. MTczODc4K1JhaHVsIFNyZWVkaGFyK3JhaHVsc3JlZWRoYXI4Nzg3QGdtYWlsLmNvbStRVUVTV


ElPTiAzNA==
What of the following Acts are correctly matched with the year in which they were passed?

Acts year

1. Hindu Marriage Act 1955

2. Hindu Succession Act 1956

3. Untouchability Act 1957

Select the code from following:

a) 1 only
b) 1 and 2
c) 2 and 3
d) All of the above
Correct Answer: B

Explanation

Solution (b)

Untouchability Act – 1955

For the eradication of untouchability the Untouchability Offences Act was passed by Indian
Government in 1955 in which any person forcing the disabilities of untouchability can be
sentenced to six months imprisonment or a fine of Rs. 500/- or both for his first offence. For
every subsequent offence the sentence will include both a term in jail as well as fine. If
considered necessary, the punishment can also be increased.

QUESTION 36. MTczODc4K1JhaHVsIFNyZWVkaGFyK3JhaHVsc3JlZWRoYXI4Nzg3QGdtYWlsLmNvbStRVUVTV


ElPTiAzNQ==
Who coined the word ‘Khalistan’ referring to an independent Sikh State, in 1971?

a) Master Tara Singh


b) Jagjeet Singh Chauhan
c) Professor Kapur Singh
d) Sardar Baldev Singh
Correct Answer: B

Explanation

Solution (b)

Word 'Khalistan' was first used by Jagjit Singh Chauhan in UK in 1971 when he arrived from
Pakistan with a beer of Guru Granth Sahib he obtained from Nankana Sahib with the assistance

IASbaba
Web: http://ilp.iasbaba.com/ Score:
Email: ilp@iasbaba.com 0.00 / 150
Page 25
Exam Title : 2019 - Test 19- History & ...
Email : rahulsreedhar8787@gmail.com
Contact :

of Pakistan authorities during the confrontation between India and Pakistan prior to Bangladesh
war. He arrived in Southall UK but he was not allowed to enter the Gurdwara Sahib in Havelock
Rd in view of him being perceived as an agent of Pakistan govt. He even advertised in a Punjabi
weekly from Southall a full page ad for 'Khalistan Airways' in which a PIA plane (PIA crudely
painted over but visible) was shown painted yellow and the name Khalistan Airways showing.
The term was obviously coined for a separate state for Sikhs. Idea of a similar state was earlier
voiced by Master Tara Singh during early sixties but he called it 'Sikh Homeland' to counter the
rising influence of Sant Fateh Singh. A Sikh Homeland front was formed to fight elections but
the front lost heavily.

QUESTION 37. MTczODc4K1JhaHVsIFNyZWVkaGFyK3JhaHVsc3JlZWRoYXI4Nzg3QGdtYWlsLmNvbStRVUVTV


ElPTiAzNg==
What was the name of Press Trust of India before 1949, when it was owned and operated by
Reuters?

a) Inter Press Service


b) Associated Press of India
c) United News Information
d) Press bureau of India
Correct Answer: B

Explanation

Solution (b)

Press Trust of India (PTI) is the largest news agency in India. It is headquartered in New Delhi
and is a nonprofit cooperative among more than 500 Indian newspapers and has more than
1,000 full-time employees, as of January 22, 2016.

It took over the operations of the Associated Press of India from Reuters after India's
independence in 1947. 173878

QUESTION 38. MTczODc4K1JhaHVsIFNyZWVkaGFyK3JhaHVsc3JlZWRoYXI4Nzg3QGdtYWlsLmNvbStRVUVTV


ElPTiAzNw==
Jana Gana Mana is the National Anthem of India. Which of the following statements are correct
regarding National Anthem of India?

1. It was originally composed as Bharoto Bhagyo Bidhata in Bengali by poet Rabindranath


Tagore.

2. It was first publicly sung on 27 December 1911 at the Calcutta Session of the Indian National
Congress.

Select the code from following:

a) 1 only
b) 2 only
c) Both 1 and 2

IASbaba
Web: http://ilp.iasbaba.com/ Score:
Email: ilp@iasbaba.com 0.00 / 150
Page 26
Exam Title : 2019 - Test 19- History & ...
Email : rahulsreedhar8787@gmail.com
Contact :

d) Neither 1 nor 2
Correct Answer: C

Explanation

Solution (c)

"Jana Gana Mana" is the national anthem of India. It was originally composed as Bharoto
Bhagyo Bidhata in Bengali by poet Rabindranath Tagore. The first stanza of the song Bharoto
Bhagyo Bidhata was adopted by the Constituent Assembly of India as the National Anthem on
24 January 1950. A formal rendition of the national anthem takes approximately fifty-two
seconds. A shortened version consisting of the first and last lines (and taking about 20 seconds
to play) is also staged occasionally. It was first publicly sung on 27 December 1911 at the
Calcutta (now, Kolkata) Session of the Indian National Congress.

QUESTION 39. MTczODc4K1JhaHVsIFNyZWVkaGFyK3JhaHVsc3JlZWRoYXI4Nzg3QGdtYWlsLmNvbStRVUVTV


ElPTiAzOA==
Who became the first Chief Justice of India when the Supreme Court of India was inaugurated

on January 28 th , 1950?
a) Hiralal J Kania
b) Elijah Impey
c) Nandlal Bose
d) G V Mavalankar
Correct Answer: A

Explanation

Solution (a)

Justice Harilal Jekisundas Kania was the first Chief justice of India after the Republic Of
India in 1950. He was an acting judge in Bombay High Court in 1930, then was appointed as
Additional Judge in June 1931- March 1933. Kania succeeded Spens(Patrick Spens) on 14
August 1947.

QUESTION 40. MTczODc4K1JhaHVsIFNyZWVkaGFyK3JhaHVsc3JlZWRoYXI4Nzg3QGdtYWlsLmNvbStRVUVTV


ElPTiAzOQ==
Which of the following statements are correct regarding ‘Bhoodan Movement’?

1. It was a voluntary land reform movement in India started by Acharya Vinoba Bhave.

2. It started at Pochampally village in Telangana which is now known as Bhoodan Pochampally.

3. V Ramachandra Reddy got his title "Bhoodan" as he was the first donor of land.

Select the code from following:

a) 1 and 2

IASbaba
Web: http://ilp.iasbaba.com/ Score:
Email: ilp@iasbaba.com 0.00 / 150
Page 27
Exam Title : 2019 - Test 19- History & ...
Email : rahulsreedhar8787@gmail.com
Contact :

b) 2 and 3
c) 1 and 3
d) All of the above
Correct Answer: D

Explanation

Solution (d)

The Bhoodan Movement or Land Gift Movement, was a voluntary land reform movement in
India, started by Acharya Vinoba Bhave in 1951 at Pochampally village in Telangana which is
now known as Bhoodan Pochampally.

The mission of the movement was to persuade wealthy landowners to voluntarily give a
percentage of their land to the landless people. However, this land could not be sold. In effect,
landless labourers were being given a small plot of land on which they can settle, as well as
grow some of their own food. The Government of various Provinces, passed Bhoodan Acts which
generally stipulated that the beneficiary had no right to sell the land or use it for a non-
agricultural purpose- including forestry.

Shri Vedre Ramachandra Reddy Bhoodan got his title "Bhoodan" as he was the first donor and
initiator of the great land donation movement in early 1950s (18 April 1951) in Andhra Pradesh
(now Telangana) at a village called Pochampally in the Nalgonda district of Telangana.

QUESTION 41. MTczODc4K1JhaHVsIFNyZWVkaGFyK3JhaHVsc3JlZWRoYXI4Nzg3QGdtYWlsLmNvbStRVUVTV


lPTiA0MA==
Who was the first Indian to get Lenin Peace Price in 1952?

a) Jawaharlal Nehru
b) Saifuddin Kitchlu
c) C Rajagopalachari
d) Vijayalaxmi Pandit
Correct Answer: B

Explanation

Solution (b)

Saifuddin Kitchlew (15 January 1888 – 9 October 1963) was an Indian freedom fighter,
barrister and politician. A member of Indian National Congress, he first became Punjab Pradesh
Congress Committee (Punjab PCC) head and later the General Secretary of the AICC in 1924.
He is most remembered for the protests in Punjab after the implementation of Rowlatt Act in
March 1919, after which on 10 April, he and another leader Satya Pal, were secretly sent to
Dharamsala. A public protest rally against their arrest and that of Gandhi, on 13 April 1919 at
Jallianwala Bagh, Amritsar, led to the infamous Jallianwala Bagh massacre. He was awarded the
Stalin Peace Prize (now known as Lenin Peace Prize) in 1952.

IASbaba
Web: http://ilp.iasbaba.com/ Score:
Email: ilp@iasbaba.com 0.00 / 150
Page 28
Exam Title : 2019 - Test 19- History & ...
Email : rahulsreedhar8787@gmail.com
Contact :

QUESTION 42. MTczODc4K1JhaHVsIFNyZWVkaGFyK3JhaHVsc3JlZWRoYXI4Nzg3QGdtYWlsLmNvbStRVUVTV


lPTiA0MQ==
In which of the following sports did India win its first Olympic medal after independence?

a) Hockey
b) Wrestling
c) Boxing
d) Shooting
Correct Answer: A

Explanation

Solution (a)

The Indian field hockey team defeated the British team to win the country's only gold medal at
the 1948 Summer Olympics. It was the country's first Olympic gold medal since India became
independent.

K D Yadav won a Bronze medal in the Helsinki Olympic of 1952 in Wrestling. It was India's first
individual medal.

QUESTION 43. MTczODc4K1JhaHVsIFNyZWVkaGFyK3JhaHVsc3JlZWRoYXI4Nzg3QGdtYWlsLmNvbStRVUVTV


ElPTiA0Mg==
Which of the following statements regarding Swatantra Party are correct?

1. It was founded by Dr B R Ambedkar.

2. It was founded as a reaction to Indian National Congress as it was believed to be a


representative of only upper caste Hindus.

Select the code from following:

a) 1 only
b) 2 only
c) Both 1 and 2
d) Neither 1 nor 2
Correct Answer: D

Explanation

Solution (d)

The Swatantra Party was an Indian Classical liberal political party that existed from 1959 to
1974. It was founded by C. Rajagopalachari in reaction to what he felt was the Jawaharlal
Nehru-dominated Indian National Congress's increasingly socialist and statist outlook.

The provocation was the left turn which the Congress took at Avadi and the Nagpur
Resolutions. Swatantra (Freedom) stood for a market-based economy with the "Licence Raj"
dismantled, although it opposed laissez faire policies. The party was thus favoured by some
traders and industrialists, but at the state-level its leadership was dominated by the traditional

IASbaba
Web: http://ilp.iasbaba.com/ Score:
Email: ilp@iasbaba.com 0.00 / 150
Page 29
Exam Title : 2019 - Test 19- History & ...
Email : rahulsreedhar8787@gmail.com
Contact :

privileged classes such as zamindars (feudal landlords) and erstwhile princes. Located on the
Right of the Indian political spectrum Swatantra was not a communal party; its membership
was not restricted on the basis of religion, unlike the Hindu-nationalist Bharatiya Jana Sangh. In
1960, Rajagopalachari and his colleagues drafted a 21-point manifesto detailing why Swatantra
had to be formed, even though they were hitherto Congressmen and associates of Nehru during
the struggle for independence. The Prime Minister was highly critical of Swatantra, dubbing the
party as belonging to "the middle ages of lords, castles and zamindars".

QUESTION 44. MTczODc4K1JhaHVsIFNyZWVkaGFyK3JhaHVsc3JlZWRoYXI4Nzg3QGdtYWlsLmNvbStRVUVTV


lPTiA0Mw==
Which of the following statements are correct regarding India’s first General Elections?

1. It was held in 1947 after India got independence.

2. CPI was the party with second highest seats after Congress.

3. Sukumar Sen was the first Chief Election Commissioner of India

Select the code from following:

a) 1 and 2
b) 2 and 3
c) 1 and 3
d) All of the above
Correct Answer: B

Explanation

Solution (b)

General elections to the first Lok Sabha since independence were held in India between 25
October 1951 and 21 February 1952.

Some key things about the first general elections in India:

· The first general elections were held between October 25, 1951 and March 27, 1952

· About 1874 candidates and 53 parties contested for the elections

· The parties contested for 489 seats

· INC won the elections with 364 seats as the people voted for the party which was led by
Jawaharlal Nehru

· CPI is the party which finished second with 16 seats as they got about 3.29 per cent votes

· SOC finished third in the elections with 10.59 per cent votes and winning 12 seats

· Approximately 45 per cent of the total votes were polled for INC in the first Lok Sabha
elections

· The population of India was 36 crore, out of which, 17.32 crore population was eligible to vote

IASbaba
Web: http://ilp.iasbaba.com/ Score:
Email: ilp@iasbaba.com 0.00 / 150
Page 30
Exam Title : 2019 - Test 19- History & ...
Email : rahulsreedhar8787@gmail.com
Contact :

· The voter turnout for the elections was 45.7 per cent.

Sukumar Sen was an Indian civil servant who was the first Chief Election Commissioner of
India, serving from 21 March 1950 to 19 December 1958.

QUESTION 45. MTczODc4K1JhaHVsIFNyZWVkaGFyK3JhaHVsc3JlZWRoYXI4Nzg3QGdtYWlsLmNvbStRVUVTV


ElPTiA0NA==
Which was the first state to be formed on linguistic basis after independence?

a) Tamil nadu
b) Andhra Pradesh
c) Gujarat
d) Jammu and Kashmir
Correct Answer: B

Explanation

Solution (b)

Andhra State was a state in India created 1953, from the Telugu-speaking northern districts of
Madras State. The state was made up of two distinct cultural regions – Rayalaseema and
Coastal Andhra. It is to be noted that Andhra State was created in 1953 but it hadn't included a
few Telugu speaking areas in from Hyderabad State. After the state reorganisation act 1956,
Andhra Pradesh became the first state in India under the State reorganisation act in 1956.

In an effort to protect the interests of the Telugu people of Madras State, Potti Sreeramulu
attempted to force the Madras state government to listen to public demands for the separation
of Telugu-speaking districts (Rayalaseema and Coastal Andhra) from Madras State to form
Andhra State. He went on a lengthy fast, and only stopped when Prime Minister Jawaharlal
Nehru promised to form Andhra State. However, there was no movement on the issue for a long
time. He started fasting again for attaining statehood for Andhra in Maharshi Bulusu
Sambamurti's house in Madras on 19 October 1952. It started off without fanfare but steadily
caught people's attention despite the disapproval of the fast by the Andhra INC committee.

The government of the day, however, did not make a clear statement about the formation of a
new state, despite several strikes and demonstrations by Telugu people. On the midnight of 15
December (i.e. early 16 December 1952), Sreeramulu died trying to achieve his objective at
House No. 126, Royapethah High Road, Mylapore, Madras State; the house has been preserved
as a monument by the state government.

QUESTION 46. MTczODc4K1JhaHVsIFNyZWVkaGFyK3JhaHVsc3JlZWRoYXI4Nzg3QGdtYWlsLmNvbStRVUVTV


ElPTiA0NQ==
Which of the following statements are correct regarding ‘Mandal Commission’?

1. Its mandate was to "identify the socially or educationally backward classes" of India.

IASbaba
Web: http://ilp.iasbaba.com/ Score:
Email: ilp@iasbaba.com 0.00 / 150
Page 31
Exam Title : 2019 - Test 19- History & ...
Email : rahulsreedhar8787@gmail.com
Contact :

2. The Commission's report recommended that members of Other Backward Classes (OBC) be
granted reservations to 27 per cent of jobs under the Central government and public sector
undertakings, thus making the total number of reservations for SC, ST and OBC to 49%.

3. It was established by Rajiv Gandhi in 1989.

Select the code from following:

a) 1 and 2
b) 2 and 3
c) 1 and 3
d) All of the above
Correct Answer: A

Explanation

Solution (a)

The Mandal Commission , or the Socially and Educationally Backward Classes Commission
(SEBC), was established in India on 1 January 1979 by the Janata Party government under
Prime Minister Morarji Desai with a mandate to "identify the socially or educationally backward
classes" of India. It was headed by the late B.P. Mandal an Indian parliamentarian, to consider
the question of reservations for people to redress caste discrimination, and used eleven social,
economic, and educational indicators to determine backwardness. In 1980, based on its
rationale that OBCs ("Other backward classes") identified on the basis of caste, economic and
social indicators comprised 52% of India's population, the Commission's report recommended
that members of Other Backward Classes (OBC) be granted reservations to 27 per cent of jobs
under the Central government and public sector undertakings, thus making the total number of
reservations for SC, ST and OBC to 49%.

Though the report had been completed in 1983, the V.P. Singh government declared its intent to
implement the report in August 1990, leading to widespread student protests. It was thereafter
provided a temporary stay order by the Supreme Court, but implemented in 1992 in the central
government.

QUESTION 47. MTczODc4K1JhaHVsIFNyZWVkaGFyK3JhaHVsc3JlZWRoYXI4Nzg3QGdtYWlsLmNvbStRVUVTV


ElPTiA0Ng==
What was the code name given to first Nuclear Test – Pokhran I?

a) Laughing Buddha
b) Sleeping Buddha
c) Smiling Buddha
d) Bodhisatva
Correct Answer: C
Explanation

Solution (c)

IASbaba
Web: http://ilp.iasbaba.com/ Score:
Email: ilp@iasbaba.com 0.00 / 150
Page 32
Exam Title : 2019 - Test 19- History & ...
Email : rahulsreedhar8787@gmail.com
Contact :

Smiling Buddha (MEA designation: Pokhran-I) was the assigned code name of India's first
successful nuclear bomb test on 18 May 1974. The bomb was detonated on the army base,
Pokhran Test Range (PTR), in Rajasthan by the Indian Army under the supervision of several
key Indian generals.

Pokhran-I was also the first confirmed nuclear weapons test by a nation outside the five
permanent members of the United Nations Security Council. Officially, the Indian Ministry of
External Affairs (MEA) claimed this test was a "peaceful nuclear explosion".

QUESTION 48. MTczODc4K1JhaHVsIFNyZWVkaGFyK3JhaHVsc3JlZWRoYXI4Nzg3QGdtYWlsLmNvbStRVUVTV


ElPTiA0Nw==
Which of the following statements regarding ‘Total Revolution’ in India are correct?

1. It was started by Deen Dayal Upadyay.

2. It was a combination of several revolutions to bring about a change in society which is in


tune with the ideals of Sarvodaya.

Select the code from following:

a) 1 only
b) 2 only
c) Both 1 and 2
d) Neither 1 nor 2
Correct Answer: B

Explanation

Solution (b)

Jayaprakash Narayan’s Concept of Total Revolution:

Jaya Prakash once stated that Total Revolution is a combination of seven revolutions, viz.,
political, social, economic, cultural, ideological or intellectual, educational and spiritual; and the
main motive being to bring in a change in the existing society that is in tune with the ideals of
the Sarvodaya. JP had a very idealistic notion of society and it is in this endeavor, he shifted
from Marxism to Socialism and later towards Sarvodaya.

By the early 1970s, JP completely withdrew from party and power politics, and concentrated
more on social regeneration through peaceful means. This did not mean that JP kept quiet while
there was social and political degeneration taking root in political freedom. In order to better
the situation, despite his old age, he embarked on the task of working towards bringing in a
complete change in the political and economic life of India.

QUESTION 49. MTczODc4K1JhaHVsIFNyZWVkaGFyK3JhaHVsc3JlZWRoYXI4Nzg3QGdtYWlsLmNvbStRVUVTV


ElPTiA0OA==
In which of the following years did India fight wars with Pakistan?

1. 1948

IASbaba
Web: http://ilp.iasbaba.com/ Score:
Email: ilp@iasbaba.com 0.00 / 150
Page 33
Exam Title : 2019 - Test 19- History & ...
Email : rahulsreedhar8787@gmail.com
Contact :

2. 1961

3. 1971

4. 1999

Select the code from following:

a) 1,2 and 3
b) 2,3 and 4
c) 1,3 and 4
d) All of the above
Correct Answer: C

Explanation

Solution (c)

Since the partition of British India in 1947 and creation of modern states of India and Pakistan,
the two South Asian countries have been involved in four wars, including one undeclared war,
and many border skirmishes and military stand-offs.

1948 war – ended in cease fire at Line of Control in Kashmir

1965 war – ended at Tashkent Agreement

1971 war – ended with unconditional surrender of Pakistan in East Pakistan (Bangladesh)

1999 – Kargil war

QUESTION 50. MTczODc4K1JhaHVsIFNyZWVkaGFyK3JhaHVsc3JlZWRoYXI4Nzg3QGdtYWlsLmNvbStRVUVTV


ElPTiA0OQ==
Which of the following statements are correct regarding Dravida Munnetra Kazhagam (DMK)?

1. It was founded by founded by C. N. Annadurai in 1949 as a breakaway faction from the


Dravidar Kazhagam headed by Periyar E. V. Ramasamy.

2. DMK was headed by M. Karunanidhi from 1969 until his death on 7 August 2018.

Select the code from following:

a) 1 only
b) 2 only
c) Both 1 and 2
d) Neither 1 nor 2
Correct Answer: C
Explanation

Solution (c)

IASbaba
Web: http://ilp.iasbaba.com/ Score:
Email: ilp@iasbaba.com 0.00 / 150
Page 34
Exam Title : 2019 - Test 19- History & ...
Email : rahulsreedhar8787@gmail.com
Contact :

Dravida Munnetra Kazhagam (DMK) (English: Dravidian Progress Federation) is an Indian


state political party in the state of Tamil Nadu and Union Territory of Puducherry. It is a
Dravidian party founded by C. N. Annadurai in 1949 as a breakaway faction from the Dravidar
Kazhagam (known as Justice Party until 1944) headed by Periyar E. V. Ramasamy. DMK was
headed by M. Karunanidhi from 1969 until his death on 7 August 2018. He served as Chief
Minister of Tamil Nadu five times. DMK was the first party other than the Indian National
Congress to win state-level elections with a clear majority on its own in any state in India.

Over the years, many disagreements arose between Periyar and his followers. In 1949, several
of his followers led by C. N. Annadurai, decided to split from Dravidar Kazhagham, after an
aged Periyar married a young woman Maniammai and appointed his young wife to act as his
successor to lead the party, superseding senior party leaders. Until then E. V. K. Sampath, the
nephew of Periyar, was considered his political heir.

Annadurai, on 17 September 1949 along with Kudanthai K.K.Neelamegam, V. R.


Nedunchezhiyan, K. A. Mathiazhagan, N. V. Natarajan called "Aimberum Thalaivargal" (Great
Five Leaders) along with M. Karunanidhi then an emerging screenwriter and thousands of
others in Robinson park in Royapuram in Chennai announced the formation of the DMK. The
name of the party (DMK) was announced by Kudanthai Perunthagai. K. K. Neelamegam.

QUESTION 51. MTczODc4K1JhaHVsIFNyZWVkaGFyK3JhaHVsc3JlZWRoYXI4Nzg3QGdtYWlsLmNvbStRVUVTV


lPTiA1MA==
Consider the following statements with respect to ‘‘Yuva Sahakar’ Scheme

1. It provides for a variety of interventions for universal access and retention in elementary
education

2. It is under the aegis of Ministry of Human Resource Development

3. It includes opening of new schools and alternate schooling facilities

Select the correct statements

a) 1 and 2
b) 2 and 3
c) All of the above
d) None of the above
Correct Answer: D

Explanation

Solution (d)

‘Yuva Sahakar-Cooperative Enterprise Support and Innovation Scheme’

· It is launched by National Cooperative Development Corporation (NCDC)

· It was launched to encourage youth to join cooperative business ventures in agriculture and
allied sectors.

IASbaba
Web: http://ilp.iasbaba.com/ Score:
Email: ilp@iasbaba.com 0.00 / 150
Page 35
Exam Title : 2019 - Test 19- History & ...
Email : rahulsreedhar8787@gmail.com
Contact :

· It will be linked to Rs 1000 crore ‘Cooperative Start-up and Innovation Fund (CSIF) created by
the NCDC.

· It would have more incentives for cooperatives of North Eastern region, aspirational districts
and cooperatives with women members, schedule castes, schedule tribes or people with
disabilities.

· Apart from a subsidy for the entrepreneur, the scheme also makes available an interest
subvention of 2% per annum.

· The scheme envisages 2% less than the applicable rate of interest on term loans for the
project cost of up to Rs 3 crore including 2 years moratorium on payment of principal. All types
of cooperatives in operation for at least one year are eligible

NCDC

· It is a statutory organisation functioning as an apex financial and developmental institution


which supports cooperatives in diverse fields apart from agriculture and allied sectors.

QUESTION 52. MTczODc4K1JhaHVsIFNyZWVkaGFyK3JhaHVsc3JlZWRoYXI4Nzg3QGdtYWlsLmNvbStRVUVTV


lPTiA1MQ==
Consider the following statements with respect to ‘National Cooperative Development
Corporation (NCDC)’

1. It is engaged in the development of Agriculture sector through cooperatives.

2. It is a statutory Corporation under the Ministry of Agriculture & Farmers Welfare.

Select the correct statements

a) 1 Only
b) 2 Only
c) Both 1 and 2
d) Neither 1 nor 2
Correct Answer: C

Explanation

Solution (c)

NCDC is a statutory Corporation set up under an Act of Parliament on 13th March 1963.

It is under the Ministry of Agriculture & Farmers Welfare.

The objectives of NCDC are planning and promoting programmes for production, processing,
marketing, storage, export and import of agricultural produce, foodstuffs, industrial goods,
livestock and certain other notified commodities and services on cooperative principles and for
matters concerned therewith or incidental thereto.

NCDC promotes and provides financial assistance to cooperatives for following


activities:

IASbaba
Web: http://ilp.iasbaba.com/ Score:
Email: ilp@iasbaba.com 0.00 / 150
Page 36
Exam Title : 2019 - Test 19- History & ...
Email : rahulsreedhar8787@gmail.com
Contact :

· Marketing & Inputs

· Agro-processing viz. Sugar, Spinning Mills, Ginning & Pressing, Fruit & Vegetables, other
small/ medium sized processing units

· Storage

· Cold Storage

· Consumer

· Weaker Sections Viz. Tribal, Fisheries, Dairy, Poultry, Sericulture, Handloom, Coir and Jute

· Power loom

· Equipment Finance

· Computerisation

NCDC raises funds from Central Govt. (DOAC&FW, SDF), Internal Accruals, market borrowings,
funding from multinational organisations like World Bank, EEC etc.

QUESTION 53. MTczODc4K1JhaHVsIFNyZWVkaGFyK3JhaHVsc3JlZWRoYXI4Nzg3QGdtYWlsLmNvbStRVUVTV


ElPTiA1Mg==
Consider the following statements with respect to ‘Bhoomi-Rashi portal’

1. It allows for totally digital and paper-less processing of land acquisition cases

2. It is developed by Ministry of Road Transport & Highways

Select the correct statements

a) 1 Only
b) 2 Only
c) Both 1 and 2
d) Neither 1 nor 2
Correct Answer: C
Explanation

Solution (c)

Bhoomi-Rashi portal

· It allows for totally digital and paper-less processing of land acquisition cases, and has
resulted in transparent, quick, corruption-free and error-free handling of land acquisition cases.

· It has also made the possible real-time tracking of activities and generation of reports relating
to land acquisition.

· It is developed by Ministry of Road Transport & Highways

IASbaba
Web: http://ilp.iasbaba.com/ Score:
Email: ilp@iasbaba.com 0.00 / 150
Page 37
Exam Title : 2019 - Test 19- History & ...
Email : rahulsreedhar8787@gmail.com
Contact :

· With this portal the approved compensation amount is directly transferred into the account of
the person concerned, through Public Financial Management System which is an integral part
of the portal.

QUESTION 54. MTczODc4K1JhaHVsIFNyZWVkaGFyK3JhaHVsc3JlZWRoYXI4Nzg3QGdtYWlsLmNvbStRVUVTV


lPTiA1Mw==
Consider the following statements with respect to ‘Sovereign Guarantee’

1. It is a promise by the Government to discharge the liability of a third person in case of his
default.

2. Article 292 of the Constitution of India extends the executive power of the Union to the
giving of guarantees on the security of the Consolidated Fund of India

Select the correct statements

a) 1 Only
b) 2 Only
c) Both 1 and 2
d) Neither 1 nor 2
Correct Answer: C

Explanation

Solution (c)

Sovereign Guarantee is a promise by the Government to discharge the liability of a third person
in case of his default.

Sovereign Guarantees are contingent liabilities of the Central and State Governments that come
into play on the occurrence of an event covered by the guarantee.

The guarantee cover of the Government of India (GoI) is limited only to the payment of principal
and normal interest in case of default. GoI is not be liable to pay any penal interest/any other
charges. Further, in view of the quasi-sovereign nature of the borrowings, it is stipulated that
the interest payable should compare with yield on G-securities of comparable maturity with a
small spread. The guarantee once given would not be transferable to any other agency. In case
of default, the lending agency has to invoke the Guarantee within a time limit of 45 to 90 days
of the default. In case the guarantee is not invoked within that stipulated period, the guarantee
would cease to exist for that portion of the tranche/loan/liability for which guarantee has not
been invoked.

Article 292 of the Constitution of India extends the executive power of the Union to the giving
of guarantees on the security of the Consolidated Fund of India, within such limits, if any, as
may be fixed by Parliament. Similar powers are given to States under Article 293.

The sovereign guarantee is normally extended for the purpose of achieving the following
objectives:-

· To improve viability of projects or activities with significant social and economic benefits,
undertaken by government or non-government entities under Public Private Partnerships;

IASbaba
Web: http://ilp.iasbaba.com/ Score:
Email: ilp@iasbaba.com 0.00 / 150
Page 38
Exam Title : 2019 - Test 19- History & ...
Email : rahulsreedhar8787@gmail.com
Contact :

· To enable public sector companies to raise resources at lower interest charges or on more
favourable terms;

· To fulfill the requirement in cases where sovereign guarantee is a precondition for


concessional loans from bilateral/multilateral agencies to sub-sovereign borrowers.

QUESTION 55. MTczODc4K1JhaHVsIFNyZWVkaGFyK3JhaHVsc3JlZWRoYXI4Nzg3QGdtYWlsLmNvbStRVUVTV


ElPTiA1NA==
‘Matua’, a Hindu Sect primarily lives in which of the following states?

a) Kerala
b) Uttarakhand
c) Jammu and Kashmir
d) West Bengal
Correct Answer: D
Explanation

Solution (d)

Matua

· It is a sect of Hindu folk religion

· The Matuas, with an estimated population of nearly 30 lakh in the state, mostly live in Nadia,
North 24 Parganas, South 24 Parganas, Kolkata and Hooghly districts of the state.

· Harichand Thakur, a Hindu monk, is known to have worked for uplifting the untouchables in
Bengal.

· He founded the Matua sect, mostly comprising members of the Namasudra community.

Matua Mahasangha or MMS

· It is a religious reformation movement that originated in what is today Bangladesh, with a


considerable number of adherents both in Bangladesh as well as in West Bengal.

· The movement was launched as a reformation by the followers of Harichand Thakur, popularly
to by adherents as God Harichand.

· Born to a peasant family belonging to the Namasudra community, Thakur attained


atmadarshan at an early age and would subsequently preach his Darshan in Twelve
Commandments.

· The teachings of Thakur establish education as preeminently important for the adherent and
the upliftment of the population the adherent's duty, while also providing a formula for ending
social conflict.

· The Matua movement originated at Orakandi, a village in Faridpur district in the mid-19th
Century under the leadership of Harichand Thakur. After Harichand’s death in 1878, his son
Guruchand Thakur provided a boost to the movement and expanded its influence over a large
section of the community.

IASbaba
Web: http://ilp.iasbaba.com/ Score:
Email: ilp@iasbaba.com 0.00 / 150
Page 39
Exam Title : 2019 - Test 19- History & ...
Email : rahulsreedhar8787@gmail.com
Contact :

THINK!

· Namashudra movement

Read More - http://frontierweekly.com/archive/vol-number/vol/vol-43-2010-11/


vol-43-37/matuas-43-37.pdf

QUESTION 56. MTczODc4K1JhaHVsIFNyZWVkaGFyK3JhaHVsc3JlZWRoYXI4Nzg3QGdtYWlsLmNvbStRVUVTV


ElPTiA1NQ==
The ‘Namasudra Movement’ is associated with which of the following states?

a) West Bengal
b) Karnataka
c) Uttar Pradesh
d) Maharashtra
Correct Answer: A

Explanation

Solution (a)

Namasudra is an Indian community originating from certain regions of Bengal.

Read More - https://www.forwardpress.in/2016/11/bengals-unsung-namasudra-


movement/

QUESTION 57. MTczODc4K1JhaHVsIFNyZWVkaGFyK3JhaHVsc3JlZWRoYXI4Nzg3QGdtYWlsLmNvbStRVUVTV


ElPTiA1Ng==
Consider the following statements with respect to ‘Network for Development of Agricultural
Cooperatives in Asia and the Pacific (NEDAC)’

1. It was set up by the Food and Agriculture Organisation (FAO)

2. It is a mix of Government and non-Government organizations

Select the correct statements

a) 1 Only
b) 2 Only
c) Both 1 and 2
d) Neither 1 nor 2
Correct Answer: C
Explanation

Solution (c)

IASbaba
Web: http://ilp.iasbaba.com/ Score:
Email: ilp@iasbaba.com 0.00 / 150
Page 40
Exam Title : 2019 - Test 19- History & ...
Email : rahulsreedhar8787@gmail.com
Contact :

Network for Development of Agricultural Cooperatives in Asia and the Pacific (NEDAC)

· It is a unique organization encompassing mix of Government and non-Government


organizations created by FAO for synergizing policies and programmes of government and
cooperative institutions at country level.

· NEDAC was set up in 1991 by the United Nations’ Food and Agriculture Organisation (FAO),
the International Cooperative Alliance (ICA) and the International Labour Organisation (ILO).

· NEDAC sensitises Governments in the region on the role of agricultural cooperatives in


promoting agricultural and rural development to ensure rural food and livelihood security for
millions of people in Asia and Pacific.

QUESTION 58. MTczODc4K1JhaHVsIFNyZWVkaGFyK3JhaHVsc3JlZWRoYXI4Nzg3QGdtYWlsLmNvbStRVUVTV


ElPTiA1Nw==
Consider the following statements with respect to ‘Himalayan State Regional Council’

1. It will be chaired by the Minister of Development of North Eastern Region

2. It will review and implement identified action points based on the reports of five working
groups which have been tasked to prepare a roadmap for revival of springs in Himalayas for
water security, monitor tourism sector standards and strengthen skill & entrepreneurship with
focus on identified priority sectors.

Select the correct statements

a) 1 Only
b) 2 Only
c) Both 1 and 2
d) Neither 1 nor 2
Correct Answer: B
Explanation

Solution (b)

Himalayan State Regional Council

· NITI Aayog has constituted the ‘Himalayan State Regional Council’ to ensure sustainable
development of the Indian Himalayan region.

· The Council has been constituted to review and implement identified action points based on
the Reports of five Working Groups, which were established along thematic areas to prepare a
roadmap for action.

· It will be the nodal agency for the Sustainable development in the Himalayan Region which
consists of the twelve States namely Jammu &Kashmir, Uttarakhand, Himachal Pradesh,
Arunachal Pradesh, Manipur, Meghalaya, Mizoram, Nagaland, Sikkim, Tripura, two districts of
Assam namely Dima Hasao and KarbiAnglong and Darjeeling and Kalimpong in West Bengal.

IASbaba
Web: http://ilp.iasbaba.com/ Score:
Email: ilp@iasbaba.com 0.00 / 150
Page 41
Exam Title : 2019 - Test 19- History & ...
Email : rahulsreedhar8787@gmail.com
Contact :

· It will be chaired by the Dr VK Saraswat, Member, NITI Aayog and will consist of the Chief
Secretaries of the Himalayan States as well as the Secretaries of key Central Ministries, senior
officers of NITI Aayog as well as special invitees.

· Five working groups have been tasked to prepare a roadmap for revival of springs in
Himalayas for water security, monitor tourism sector standards and strengthen skill &
entrepreneurship with focus on identified priority sectors.

QUESTION 59. MTczODc4K1JhaHVsIFNyZWVkaGFyK3JhaHVsc3JlZWRoYXI4Nzg3QGdtYWlsLmNvbStRVUVTV


ElPTiA1OA==
Consider the following statements with respect to ‘Lokayukta’

1. It is an anti-corruption authority constituted at the state level.

2. It came into existence after the Second Administrative Reforms Commission (ARC)
recommendations

Select the correct statements

a) 1 Only
b) 2 Only
c) Both 1 and 2
d) Neither 1 nor 2
Correct Answer: A
Explanation

Solution (a)

Lokayukta

· It is an anti-corruption authority constituted at the state level.

· It investigates allegations of corruption and mal-administration against public servants and is


tasked with speedy redressal of public grievances.

· The first Administrative Reforms Commission headed by Mr. Morarjee Desai recommended for
Sweden based Ombudsmen type of institution. Accordingly the centre can have Lokpal and
states can have Lokayukta. India does not have Lokpal yet.

· Once appointed, Lokayukta cannot be dismissed nor transferred by the government, and can
only be removed by passing an impeachment motion by the state assembly.

DO YOU KNOW?

· An amendment to the Constitution has been proposed to implement the Lokayukta uniformly
across Indian states.

· The proposed changes will make the institution of Lokayukta uniform across the country as a
three-member body, headed by a retired Supreme Court judge or high court chief justice and
comprising the state vigilance commissioner and a jurist or an eminent administrator as other
members.

IASbaba
Web: http://ilp.iasbaba.com/ Score:
Email: ilp@iasbaba.com 0.00 / 150
Page 42
Exam Title : 2019 - Test 19- History & ...
Email : rahulsreedhar8787@gmail.com
Contact :

THINK!

· Jan Lokpal Bill

· Corruption Perceptions Index

QUESTION 60. MTczODc4K1JhaHVsIFNyZWVkaGFyK3JhaHVsc3JlZWRoYXI4Nzg3QGdtYWlsLmNvbStRVUVTV


ElPTiA1OQ==
Consider the following statements The Maternity Benefit Act, 2017’

1. It increases the paid maternity leave from 12 weeks to 26 weeks.

2. It does not cover adopting mothers.

3. It makes crèche facility mandatory for every establishment employing 50 or more employees.

Select the correct statements

a) 1 and 2
b) 2 and 3
c) 1 and 3
d) All of the above
Correct Answer: C

Explanation

Solution (c)

The Maternity Benefit Act

News: Ministry of Labour and Employment is working on an incentive scheme wherein seven
weeks' wages would be reimbursed to employers who employ women workers with a wage
ceiling of up to Rs 15,000 and provide the maternity benefit of 26 weeks paid leave, subject to
certain conditions.

Why?

· To encourage employers, especially in the private sector, to implement the extended 26-week
maternity leave law

· There is a wide perception that private entities are not encouraging women employees
because if they are employed, they may have to provide maternity benefit to them, particularly
26 weeks of paid holiday

About

· It applies to establishments employing 10 or more than 10 persons in factories, mines,


plantation, shops & establishments and other entities.

· The main purpose of this Act is to regulate the employment of women in certain
establishments for certain period before and after child birth and to provide maternity benefit
and certain other benefits.

IASbaba
Web: http://ilp.iasbaba.com/ Score:
Email: ilp@iasbaba.com 0.00 / 150
Page 43
Exam Title : 2019 - Test 19- History & ...
Email : rahulsreedhar8787@gmail.com
Contact :

· The Act was amended through the Maternity Benefit (Amendment) Act, 2017, which, inter alia,
has increased the paid maternity leave to women employees from 12 weeks to 26 weeks.

· 26 weeks leave is only applicable for the women who have less than two surviving children.

· For women who are expecting after having 2 children, the duration of paid maternity leave
shall be 12 weeks.

· It introduced an enabling provision relating to "work from home" for women, which may be
exercised after the expiry of the 26 weeks' leave period.

· Maternity leave of 12 weeks to be available to mothers adopting a child below the age of three
months from the date of adoption as well as to the “commissioning mothers”.

· The commissioning mother has been defined as biological mother who uses her egg to create
an embryo planted in any other woman.

· It makes crèche facility mandatory for every establishment employing 50 or more employees.

QUESTION 61. MTczODc4K1JhaHVsIFNyZWVkaGFyK3JhaHVsc3JlZWRoYXI4Nzg3QGdtYWlsLmNvbStRVUVTV


lPTiA2MA==
Consider the following statements with respect to ‘National Council on India's Nutrition
Challenges’

1. It has been set up under POSHAN Abhiyaan

2. It is chaired by Union Minister of Women and Child Development

Select the correct statements

a) 1 Only
b) 2 Only
c) Both 1 and 2
d) Neither 1 nor 2
Correct Answer: A

Explanation

Solution (a)

National Council on India's Nutrition Challenges

· It has been set up under POSHAN Abhiyaan, apex body to formulate overall policies, guide and
monitor all nutrition based schemes.

· It is chaired by Vice Chairman, NITI Aayog

Mandate

· To provide policy directions to address India’s Nutrition Challenges through coordinated inter-
sectoral action

IASbaba
Web: http://ilp.iasbaba.com/ Score:
Email: ilp@iasbaba.com 0.00 / 150
Page 44
Exam Title : 2019 - Test 19- History & ...
Email : rahulsreedhar8787@gmail.com
Contact :

· To coordinate and review convergence among ministries

· To review programmes for nutrition on a quarterly basis

QUESTION 62. MTczODc4K1JhaHVsIFNyZWVkaGFyK3JhaHVsc3JlZWRoYXI4Nzg3QGdtYWlsLmNvbStRVUVTV


lPTiA2MQ==
The term ‘CAAQMS’ was in news recently. What is it related to?

a) International Solar Alliance


b) Air Quality
c) Soil Health
d) Operation Greens
Correct Answer: B

Explanation

Solution (b)

Continuous Ambient Air Quality Monitoring System (CAAQMS)

· It is used for monitoring ambient air quality on real- time basis.

· It provides continuous real-time updates on particulate matter, sulphur dioxide, and NO2
concentrations.

· It is also known as an automatic monitoring station.

· CAAQMS is installed both by the government as well as industries to monitor ambient air
quality in the respective regions.

QUESTION 63. MTczODc4K1JhaHVsIFNyZWVkaGFyK3JhaHVsc3JlZWRoYXI4Nzg3QGdtYWlsLmNvbStRVUVTV


ElPTiA2Mg==
Which of the following are examples of ‘Metabolic Syndrome (MS)’

1. Tuberculosis

2. Increased blood pressure

3. High blood sugar

Select the correct code:

a) 1 and 2
b) 2 and 3
c) 1 and 3
d) 3 Only
Correct Answer: B

IASbaba
Web: http://ilp.iasbaba.com/ Score:
Email: ilp@iasbaba.com 0.00 / 150
Page 45
Exam Title : 2019 - Test 19- History & ...
Email : rahulsreedhar8787@gmail.com
Contact :

Explanation

Solution (b)

Metabolic Syndrome (MS)

· It is a cluster of metabolic abnormalities — increased blood pressure, high blood sugar,


abdominal obesity, abnormal cholesterol or triglyceride levels — that occur together, increasing
one’s risk of heart disease, stroke and diabetes.

· Insulin resistance, metabolic syndrome, and prediabetes are closely related to one another and
have overlapping aspects.

DO YOU KNOW?

· WHO recommends that trans-fat intake be limited to less than 1% of total energy intake and
has called for the total elimination of TFAs in global food supply by 2023.

QUESTION 64. MTczODc4K1JhaHVsIFNyZWVkaGFyK3JhaHVsc3JlZWRoYXI4Nzg3QGdtYWlsLmNvbStRVUVTV


lPTiA2Mw==
Consider the following statements with respect to ‘Parambikulam Tiger Reserve’

1. It is spread over Kerala and Karnataka

2. The Onge tribe primarily live in this reserve

Select the correct statements

a) 1 Only
b) 2 Only
c) Both 1 and 2
d) Neither 1 nor 2
Correct Answer: D
Explanation

Solution (d)

Parambikulam Tiger Reserve

· It is located in Palakkad district of Kerala

· It also includes the erstwhile Parambikulam Wildlife Sanctuary

· It is the home of four different tribes of indigenous peoples including the Kadar, Malasar,
Muduvar and Mala Malasar settled in six colonies.

· The oldest ever teak tree, Kannimara Teak exists here

QUESTION 65. MTczODc4K1JhaHVsIFNyZWVkaGFyK3JhaHVsc3JlZWRoYXI4Nzg3QGdtYWlsLmNvbStRVUVTV


ElPTiA2NA==

IASbaba
Web: http://ilp.iasbaba.com/ Score:
Email: ilp@iasbaba.com 0.00 / 150
Page 46
Exam Title : 2019 - Test 19- History & ...
Email : rahulsreedhar8787@gmail.com
Contact :

Consider the following statements with respect to ‘National Cyclone Risk Mitigation Project’

1. It was started by the Ministry of Home Affairs.

2. It was started aftermath of ‘2004 Indian Ocean earthquake and tsunami’

Select the correct statements

a) 1 Only
b) 2 Only
c) Both 1 and 2
d) Neither 1 nor 2
Correct Answer: A

Explanation

Solution (a)

The Government of India has initiated the National Cyclone Risk Mitigation Project (NCRMP)
with a view to address cyclone risks in the country. The overall objective of the Project is to
undertake suitable structural and non-structural measures to mitigate the effects of cyclones in
the coastal states and UT’s of India.

Ministry of Home Affairs (MHA), Government of India conceptualized a comprehensive National


Cyclone Risk Mitigation Strategy through consultation, ending with a National workshop,
"Developing Strategy for Cyclone Mitigation in the Coastal and Island Regions of India", held on
4th and 5th of February, 2003

National Disaster Management Authority (NDMA) under the aegis of Ministry of Home Affairs
(MHA) will implement the Project in coordination with participating State Governments and the
National Institute for Disaster Management (NIDM).

The Project has identified 13 cyclone prone States and Union Territories (UTs), with varying
levels of vulnerability. These States/UT have further been classified into two categories, based
on the frequency of occurrence of cyclone, size of population and the existing institutional
mechanism for disaster management. These categories are:

· Category I: Higher vulnerability States i.e. Andhra Pradesh, Gujarat, Odisha, Tamil Nadu and
West Bengal.

· Category II: Lower vulnerability States i.e. Maharashtra, Karnataka, Kerala, Goa, Puducherry,
Lakshadweep, Daman and Diu, Andaman and Nicobar Islands.

QUESTION 66. MTczODc4K1JhaHVsIFNyZWVkaGFyK3JhaHVsc3JlZWRoYXI4Nzg3QGdtYWlsLmNvbStRVUVTV


ElPTiA2NQ==
‘Composite Water Management Index (CWMI)’ is released by

a) UNEP
b) WEF
c) World Bank
d) NITI Aayog

IASbaba
Web: http://ilp.iasbaba.com/ Score:
Email: ilp@iasbaba.com 0.00 / 150
Page 47
Exam Title : 2019 - Test 19- History & ...
Email : rahulsreedhar8787@gmail.com
Contact :

Correct Answer: D

Explanation

Solution (d)

Composite Water Management Index (CWMI) by the NITI Aayog

· It was published in association with the Ministry of Water Resources, Ministry of Drinking
Water and Sanitation and the Ministry of Rural Development.

· It shows that 600 million people face high to extreme water stress in India

· It predicts that a persistent water crisis will lead to an eventual 6% loss in the country’s Gross
Domestic Product by 2030.

QUESTION 67. MTczODc4K1JhaHVsIFNyZWVkaGFyK3JhaHVsc3JlZWRoYXI4Nzg3QGdtYWlsLmNvbStRVUVTV


ElPTiA2Ng==
Consider the following statements with respect to ‘Sri Lankan Frogmouth’

1. It is a rodent with a coat of sharp spines, or quills, that protect against predators.

2. It is only found in Sri Lanka

Select the correct statements

a) 1 Only
b) 2 Only
c) Both 1 and 2
d) Neither 1 nor 2
Correct Answer: D

Explanation

Solution (d)

Sri Lankan Frogmouth

News: It was sotted in Chinnar Wildlife Sanctuary. Its presence was noticed on the eastern side
of the Western Ghats for the first time

About

· It is found in the Western Ghats of south India and Sri Lanka.

· It is usually confined to its habitation in the western side of the Western Ghats forests

· It is a relative of Nightjar, a crepuscular and nocturnal bird breeding in Europe and temperate
Asia.

· Its preferred habitat is a dry and open area with some small trees or bushes.

· It eats insects and mainly seeks prey during night time.

IASbaba
Web: http://ilp.iasbaba.com/ Score:
Email: ilp@iasbaba.com 0.00 / 150
Page 48
Exam Title : 2019 - Test 19- History & ...
Email : rahulsreedhar8787@gmail.com
Contact :

· It lays only one egg a year after the mating season in April-May.

· The nest is made using moss or leaves of soft plants and the bark of the trees. The male bird
destroys the nest and flies away with the new born bird.

QUESTION 68. MTczODc4K1JhaHVsIFNyZWVkaGFyK3JhaHVsc3JlZWRoYXI4Nzg3QGdtYWlsLmNvbStRVUVTV


ElPTiA2Nw==
Consider the following statements and identify the ‘Wildlife Sanctuary’

1. It has earned the name for only rehabilitation centre for star tortoise in India.

2. It forms an integral part of the block of protected forests straddling the Kerala-Tamil Nadu
border in the Anaimalai Hills

3. The main inhabitants are Muthuvas and Pulayars

Select the correct code:

a) Chinnar WLS
b) Nagarjuna Sagar-Srisailam WLS
c) Bhimashankar WLS
d) Ranganathittu Bird WLS
Correct Answer: A
Explanation

Solution (a)

Chinnar Wildlife Sanctuary

· It is located in Kerala

· It has earned the name for only rehabilitation centre for star tortoise in India.

· It is under the jurisdiction of and contiguous with Eravikulam National Park to the south.
Indira Gandhi Wildlife Sanctuary is to the north and Kodaikanal Wildlife Sanctuary is to the
east.

· It forms an integral part of the block of protected forests straddling the Kerala-Tamil Nadu
border in the Anaimalai Hills.

· The Western Ghats, Anamalai Sub-Cluster, including all of Chinnar Wildlife Sanctuary, is under
consideration by the UNESCO World Heritage Committee for selection as a World Heritage
Site.

· There are 11 tribal settlements inside the Chinnar WLS, each is well demarcated by temporary
stone walls. The main inhabitants are Muthuvas and Pulayars.

IASbaba
Web: http://ilp.iasbaba.com/ Score:
Email: ilp@iasbaba.com 0.00 / 150
Page 49
Exam Title : 2019 - Test 19- History & ...
Email : rahulsreedhar8787@gmail.com
Contact :

QUESTION 69. MTczODc4K1JhaHVsIFNyZWVkaGFyK3JhaHVsc3JlZWRoYXI4Nzg3QGdtYWlsLmNvbStRVUVTV


ElPTiA2OA==
The ‘Chitmahal Agreement’ is associated with

a) Pakistan
b) Myanmar
c) Bangladesh
d) Nepal
Correct Answer: C
Explanation

Solution (c)

Enclave exchange

· Bangladesh and India had exchanged a total of 162 enclaves on August 1, 2015, ending one of
the world’s most-complex border disputes that had lingered for seven decades since
Independence.

· As per the Land Boundary Agreement (LBA) 1974 and Protocol of 2011, 51 erstwhile
Bangladeshi enclaves in India and 111 erstwhile Indian enclaves in Bangladesh were physically
transferred to the other country.

· They are also known as the chitmahals

· It settled the question of citizenship for over 50,000 people in these enclaves.

· The enclaves were part of West Bengal, Tripura, Assam and Meghalaya.

THINK!

· Indira-Mujib agreement

QUESTION 70. MTczODc4K1JhaHVsIFNyZWVkaGFyK3JhaHVsc3JlZWRoYXI4Nzg3QGdtYWlsLmNvbStRVUVTV


ElPTiA2OQ==
Consider the following statements with respect to ‘Solar Bubble Dryer’

1. It is developed by Central Mechanical Engineering Research Institute (CMERI)

2. It will help bring down quantitative and qualitative losses caused by high moisture content
during post-harvest operation

Select the correct statements

a) 1 Only
b) 2 Only
c) Both 1 and 2
d) Neither 1 nor 2
Correct Answer: B

IASbaba
Web: http://ilp.iasbaba.com/ Score:
Email: ilp@iasbaba.com 0.00 / 150
Page 50
Exam Title : 2019 - Test 19- History & ...
Email : rahulsreedhar8787@gmail.com
Contact :

Explanation

Solution (b)

Solar Bubble Dryer

· It is a low-cost drying technology that aims to provide a simple and flexible alternative to sun-
drying, while protecting from spillage, animals, weather and vehicles running over the grains

· It is jointly developed by International Rice Research Institute, Philippines; Grainpro, a leading


post-harvest solution providing company; and University of Hohenheim, Germany.

· It is a handy tool to bring down quantitative and qualitative losses caused by high moisture
content during post-harvest operation

· Farmers can dismantle the machinery and reassemble it on their own. Power can be drawn
both from solar energy and traditional electricity.

QUESTION 71. MTczODc4K1JhaHVsIFNyZWVkaGFyK3JhaHVsc3JlZWRoYXI4Nzg3QGdtYWlsLmNvbStRVUVTV


lPTiA3MA==
Which of the following statements about ‘Kambala’ is/are correct?

1. It is prevalent in all the Southern States

2. It is similar to Spanish-style bullfighting

Select the correct code:

a) 1 Only
b) 2 Only
c) Both 1 and 2
d) Neither 1 nor 2
Correct Answer: D
Explanation

Solution (d)

Kambala

· It is an annual buffalo race held in the southwestern Indian state of Karnataka.

· Traditionally, it is sponsored by local Tuluva landlords and households in the coastal districts
of Dakshina Kannada and Udupi, a region collectively known as Tulu Nadu.

· Kambala is a contest where two pairs of buffaloes tied ploughs are made to race on slush-filled
tracks with a jockey guiding them while they run.

QUESTION 72. MTczODc4K1JhaHVsIFNyZWVkaGFyK3JhaHVsc3JlZWRoYXI4Nzg3QGdtYWlsLmNvbStRVUVTV


lPTiA3MQ==

IASbaba
Web: http://ilp.iasbaba.com/ Score:
Email: ilp@iasbaba.com 0.00 / 150
Page 51
Exam Title : 2019 - Test 19- History & ...
Email : rahulsreedhar8787@gmail.com
Contact :

Which of the following statements with respect to ‘Igla-S’ is/are correct?

a) It is a 155mm, 39 calibre Ultra-Light Howitzer


b) It is jointly developed by Israel and India
c) Both (a) and (b)
d) Neither (a) nor (b)
Correct Answer: D

Explanation

Solution (d)

Igla-S Missile System

News: Russian Igla-S system has been confirmed as the lowest bidder for a mega deal to
procure shoulder fired anti-aircraft missiles for the Indian Army

· It is a Russian/Soviet man-portable infrared homing surface-to-air missile (SAM).

· The process to procure Very Short Range Air Defence (Vshorad) missiles was initiated in 2010
under the previous government and went through several rounds of trails before the Igla-S
qualified, along with two other competitors.

· The Vshorad programme to replace the Russian Igla-M systems that have been used by the
Army since the 1980s is considered critical for defence against incoming helicopters, UAVs and
ground attack aircraft.

QUESTION 73. MTczODc4K1JhaHVsIFNyZWVkaGFyK3JhaHVsc3JlZWRoYXI4Nzg3QGdtYWlsLmNvbStRVUVTV


ElPTiA3Mg==
Consider the following statements with respect to ‘Eco-Sensitive Zones (ESZs)’

1. It is notified by the Ministry of Environment, Forests and Climate Change (MoEFCC) around
Protected Areas, National Parks and Wildlife Sanctuaries.

2. The Environment (Protection) Act, 1986 has fixed the width within 20 kilometres around the
protected area

Select the correct statements

a) 1 Only
b) 2 Only
c) Both 1 and 2
d) Neither 1 nor 2
Correct Answer: A
Explanation

Solution (a)

Eco-Sensitive Zones (ESZs)

IASbaba
Web: http://ilp.iasbaba.com/ Score:
Email: ilp@iasbaba.com 0.00 / 150
Page 52
Exam Title : 2019 - Test 19- History & ...
Email : rahulsreedhar8787@gmail.com
Contact :

· They are areas notified by the Ministry of Environment, Forests and Climate Change
(MoEFCC) around Protected Areas , National Parks and Wildlife Sanctuaries.

· The purpose of declaring ESZs is to create some kind of “shock absorbers” to the protected
areas by regulating and managing the activities around such areas.

· They also act as a transition zone from areas of high protection to areas involving lesser
protection.

· The Environment (Protection) Act, 1986 does not mention the word “Eco-Sensitive Zones”.

· The width of the ESZ and type of regulation may vary from protected area to area.

· An ESZ could go up to 10 kilometres around a protected area as provided in the Wildlife


Conservation Strategy, 2002.

QUESTION 74. MTczODc4K1JhaHVsIFNyZWVkaGFyK3JhaHVsc3JlZWRoYXI4Nzg3QGdtYWlsLmNvbStRVUVTV


lPTiA3Mw==
‘The Global Education Monitoring Report’ is released by

a) OECD
b) UNESCO
c) World Bank
d) None of the above
Correct Answer: B

Explanation

Solution (b)

UNESCO global education monitoring report 2019

· The Global Education Monitoring Report is an independent annual publication.

· The GEM Report is funded by a group of governments, multilateral agencies and private
foundations and facilitated and supported by UNESCO

· The 2019 GEM Report continues its assessment of progress towards Sustainable Development
Goal 4 (SDG 4) on education and its ten targets, as well as other related education targets in
the SDG agenda.

· Its main focus is on the theme of migration and displacement.

· It presents evidence on the implications of different types of migration and displacement for
education systems

· It impacts that reforming education curricula and approaches to pedagogy and teacher
preparation can have on addressing the challenges and opportunities posed by migration and
displacement.

IASbaba
Web: http://ilp.iasbaba.com/ Score:
Email: ilp@iasbaba.com 0.00 / 150
Page 53
Exam Title : 2019 - Test 19- History & ...
Email : rahulsreedhar8787@gmail.com
Contact :

QUESTION 75. MTczODc4K1JhaHVsIFNyZWVkaGFyK3JhaHVsc3JlZWRoYXI4Nzg3QGdtYWlsLmNvbStRVUVTV


ElPTiA3NA==
Which of the following statements is/are correct?

1. Sunda Strait connects the Java Sea to the Indian Ocean

2. Karimata Strait connects the South China Sea to the Java Sea

Select the correct code:

a) 1 Only
b) 2 Only
c) Both 1 and 2
d) Neither 1 nor 2
Correct Answer: C

Explanation

Solution (c)

Karimata Strait is the wide strait that connects the South China Sea to the Java Sea, separating
the Indonesian islands of Sumatra and Borneo (Kalimantan).

Sunda Strait is the strait between the Indonesian islands of Java and Sumatra. It connects the
Java Sea to the Indian Ocean.

IASbaba
Web: http://ilp.iasbaba.com/ Score:
Email: ilp@iasbaba.com 0.00 / 150
Page 54

You might also like